Open125

math #2

ryoaqryoaq

X \coloneqq \mathbb{R}^n
\mathcal{F} \coloneqq C^\infty(\mathbb{R}, X)
x(t) \in \mathcal{F} に対して T_x\mathcal{F} \coloneqq \Gamma(\mathbb{R}, x^*TX)
L \in \Gamma(\mathcal{F} \times \mathbb{R}, T^*\mathbb{R})

L \coloneqq \frac{m}{2} |\dot{x}|^2 dt

で定める
x \in C^\infty(\mathcal{F} \times \mathbb{R}, X)
\dot{x} = \frac{\partial}{\partial t} x \in \Gamma(\mathcal{F} \times \mathbb{R}, x^*TX) \simeq C^\infty(\mathcal{F} \times \mathbb{R}, \mathbb{R}^n)
\delta L \in \Gamma(\mathcal{F} \times \mathbb{R}, T^*\mathcal{F} \otimes T^*\mathbb{R}) を計算する
\delta L = m \langle \dot{x}, \delta \dot{x} \rangle dt = m \langle \dot{x}, \frac{\partial}{\partial t}(\delta x) \rangle dt = m (\frac{\partial}{\partial t} \langle \dot{x}, \delta x \rangle - \langle \ddot{x}, \delta x \rangle) dt

\delta x \in \Omega^{1, 0}(\mathcal{F} \times \mathbb{R}, x^*TX) \simeq \Omega^{1, 0}(\mathcal{F} \times \mathbb{R}, \mathbb{R}^n)
\int_{t_0}^{t_1} L \in C^\infty(\mathcal{F})
\begin{aligned} \delta \int_{t_0}^{t_1} L &= - \int_{t_0}^{t_1} \delta L \\ &= m \int_{t_0}^{t_1} dt (\frac{\partial}{\partial t} \langle \dot{x}, \delta x \rangle - \langle \ddot{x}, \delta x \rangle) \\ &= m [\langle \dot{x}(t), \delta x(t) \rangle]_{t_0}^{t_1} - m \int_{t_0}^{t_1} dt \langle \ddot{x}(t), \delta x(t) \rangle \end{aligned}

\mathcal{M} \coloneqq \{ x \in \mathcal{F} \mid \ddot{x} = 0 \} \simeq \{ x_0 + (t - t_0) v_0 \mid (x_0, v_0) \in TX \}
\gamma(t) \coloneqq m \langle \dot{x}(t), \delta x(t) \rangle \in \Omega^1(\mathcal{F}) とおくと

\delta \int_{t_0}^{t_1} L = \gamma(t_1) - \gamma(t_0) \quad \text{on } \mathcal{M}

ryoaqryoaq

TODO
(A) 主束の接続とその曲率
(B) \mathcal{M} 上の接続付き自明 \mathbb{R} 主束たちについて
(C) \mathcal{M} \simeq TX を使って書き下す

ryoaqryoaq

(A)
\pi: P \to M: G 主束
P 上の G 同変ベクトル束の完全列

0 \to VP \simeq P \times \mathfrak{g} \to TP \to \pi^*TM \to 0

がある
左分裂 \omega \in \Omega^1(P, \mathfrak{g})^G を接続という
左分裂なので \omega(X_P) \equiv X \ (X \in \mathfrak{g})
P 上の接続全体には \Omega^1(M, P \times_G \mathfrak{g}) が単純推移的に作用する
\Omega \coloneqq d\omega + \frac{1}{2}[\omega, \omega] \in \Omega^2(P, \mathfrak{g})^G

\Omega \in \Gamma(P, \wedge^2 H^*P \otimes \mathfrak{g})^G \simeq \Omega^2(M, P \times_G \mathfrak{g}) になる
\Omega(\tilde{X}, \tilde{Y}) = -[\tilde{X}, \tilde{Y}] \text{ の } VP \text{ 成分 } = \widetilde{[X, Y]} - [\tilde{X}, \tilde{Y}] \quad (X, Y \in \Gamma(M, TM))

ryoaqryoaq

P = M \times \mathbb{R} の場合
\omega = d\theta + \gamma \quad (\gamma \in \Omega^1(M))
\Omega = d\gamma
\tilde{X} = X - \gamma(X) d\theta \quad (X \in \Gamma(M, TM))

f \in C^\infty(M)
P' \coloneqq P
P \ni (x, \theta) \mapsto (x, \theta + f(x)) \in P'\mathbb{R} 主束の同型
P 上の接続 d\theta + \gammaP' 上で

d(\theta - f) + \gamma = d\theta + (\gamma - df)

に対応する

ryoaqryoaq

(B)
dt + \gamma(t)|_\mathcal{M} は自明 \mathbb{R} 主束 \mathcal{M} \times \mathbb{R} 上の接続
これらの接続は -\int_{t_0}^{t_1} L\mathcal{M} への制限によって同型
この系の射影極限を考えれば、t に依らない \mathcal{M} 上の接続付き自明 \mathbb{R} 主束ができる
曲率は \omega(t) \coloneqq d\gamma = m \langle \delta \dot{x}(t), \delta x(t) \rangle \in \Omega^2(\mathcal{F})\mathcal{M} への制限で、t に依らない

ryoaqryoaq

(C)
\mathcal{M} \simeq \{ x_0 + (t - t_0) v_0 \mid (x_0, v_0) \in TX \} を考えれば \mathcal{M}
\gamma(t) = m \langle v_0, dx_0 + (t - t_0) dv_0 \rangle
\omega(t) = m \langle dv_0, dx_0 \rangle
\int L = \frac{m}{2} |v_0|^2 \int dt

ryoaqryoaq

X: Riemann 多様体
\mathcal{F} \coloneqq C^\infty(\mathbb{R}, X)
V \in C^\infty(X)

L \coloneqq (\frac{1}{2}|\dot{x}|^2 - V(x)) dt \in \Gamma(\mathcal{F} \times \mathbb{R}, T^*\mathbb{R})

\begin{aligned} \delta L &= (g(\dot{x}, \nabla \frac{\partial}{\partial t} x) - \langle x^{*_b}dV, \delta x \rangle) dt \\ &= (g(\dot{x}, \nabla_{\frac{\partial}{\partial t}} \delta x) - \langle x^{*_b}dV, \delta x \rangle) dt \\ &= (\frac{\partial}{\partial t} g(\dot{x}, \delta x) - g(\nabla_{\frac{\partial}{\partial t}} \dot{x}, \delta x) - \langle x^{*_b}dV, \delta x \rangle) dt \\ &= (\frac{\partial}{\partial t} g(\dot{x}, \delta x) - g(\nabla_{\frac{\partial}{\partial t}} \dot{x} + x^*\mathrm{grad}V, \delta x)) dt \end{aligned}

ただし x^{*_b}dV \in \Gamma(\mathcal{F} \times \mathbb{R}, x^*(T^*X))
\begin{aligned} \delta \int_{t_0}^{t_1} L &= - \int_{t_0}^{t_1} \delta L \\ &= \int_{t_0}^{t_1} dt (\frac{\partial}{\partial t} g(\dot{x}, \delta x) - g(\nabla_{\frac{\partial}{\partial t}} \dot{x} + x^*\mathrm{grad}V, \delta x)) \\ &= [g(\dot{x}(t), \delta x(t))]_{t_0}^{t_1} - \int_{t_0}^{t_1} dt g(\nabla_{\frac{\partial}{\partial t}} \dot{x}(t) + x(t)^*\mathrm{grad}V, \delta x(t)) \end{aligned}

\mathcal{M} \coloneqq \{ x \in \mathcal{F} \mid \nabla_{\frac{\partial}{\partial t}} \dot{x} + x^*\mathrm{grad}V = 0 \}
\mathcal{M} は局所的に TX と同型
\gamma(t) \coloneqq g(\dot{x}(t), \delta x(t)) \in \Omega^1(\mathcal{F}) を考えれば、t に依らない \mathcal{M} 上の接続付き自明 \mathbb{R} 主束ができる

ryoaqryoaq

X: Riemann 多様体
P \to X \times \mathbb{R}: 接続付き自明 \mathbb{R} 主束
P の自明化を固定すると、接続は \omega_P = d\theta + \alpha \ (\alpha \in \Omega^1(X \times \mathbb{R})) と表せる
\mathcal{F} \coloneqq C^\infty(\mathbb{R}, X)
\tilde{x} \coloneqq (x, t): \mathcal{F} \times \mathbb{R} \to X \times \mathbb{R}

L \coloneqq \frac{m}{2} |\dot{x}|^2 dt - q \tilde{x}^*\alpha \in \Omega^1(\mathcal{F} \times \mathbb{R})

\alpha = V dt + A \ (V \in C^\infty(X \times \mathbb{R}), A \in \Gamma(X \times \mathbb{R}, T^*X)) と分解すると
\begin{aligned} \delta \int_{t_0}^{t_1} L &= \delta \int_{t_0}^{t_1} dt (\frac{m}{2} |\dot{x}|^2 - q \tilde{x}^*V - q \langle \tilde{x}^{*_b}A, \dot{x} \rangle) \\ &= \int_{t_0}^{t_1} dt \delta (\frac{m}{2} |\dot{x}|^2 - q \tilde{x}^*V - q \langle \tilde{x}^{*_b}A, \dot{x} \rangle) \\ &= \int_{t_0}^{t_1} dt \{ \frac{\partial}{\partial t} (m g(\dot{x}, \delta x) - q \langle \tilde{x}^{*_b}A, \delta x \rangle) \\ &\qquad - g(m \nabla_{\frac{\partial}{\partial t}} \dot{x} + q \tilde{x}^*\mathrm{grad}_XV - q (\iota_{\dot{x}} \tilde{x}^{*_b} dA)^\sharp, \delta x) \} \\ &= [m g(\dot{x}(t), \delta x(t)) - q \langle \tilde{x}(t)^{*_b}A, \delta x(t) \rangle]_{t_0}^{t_1} \\ &\qquad - \int_{t_0}^{t_1} dt g(m \nabla_{\frac{\partial}{\partial t}} \dot{x}(t) + q \tilde{x}(t)^*(\mathrm{grad}_XV - \frac{\partial}{\partial t} A^\sharp) - q (\iota_{\dot{x}(t)} \tilde{x}(t)^{*_b} d_X A)^\sharp, \delta x(t)) \end{aligned}

\begin{aligned} \delta \langle \tilde{x}^{*_b}A, \dot{x} \rangle &= \langle \nabla \tilde{x}^{*_b}A, \dot{x} \rangle + \langle \tilde{x}^{*_b}A, \nabla \dot{x} \rangle \\ &= \langle \tilde{x}^{*_b} \nabla A, \delta x \otimes \dot{x} \rangle + (\frac{\partial}{\partial t} \langle \tilde{x}^{*_b}A, \delta x \rangle - \langle \nabla_{\frac{\partial}{\partial t}} \tilde{x}^{*_b}A, \delta x \rangle) \\ &= \frac{\partial}{\partial t} \langle \tilde{x}^{*_b}A, \delta x \rangle + \langle \tilde{x}^{*_b} \nabla A, \delta x \otimes \dot{x} - \dot{x} \otimes \delta x \rangle \\ &= \frac{\partial}{\partial t} \langle \tilde{x}^{*_b}A, \delta x \rangle + \langle \tilde{x}^{*_b} dA, \delta x \wedge \dot{x} \rangle \\ &= \frac{\partial}{\partial t} \langle \tilde{x}^{*_b}A, \delta x \rangle - \langle \iota_{\dot{x}} \tilde{x}^{*_b} dA, \delta x \rangle \end{aligned}

https://en.wikipedia.org/wiki/Musical_isomorphism

https://www.damtp.cam.ac.uk/user/tong/em/el4.pdf

https://physics.stackexchange.com/questions/416048/confusion-on-maxwells-equations-and-gauge-transformations

ryoaqryoaq

https://en.wikipedia.org/wiki/Curl_(mathematics)

X は向き付けられた 3 次元 Riemann 多様体とする

\mathrm{rot}: \Gamma(TX) \simeq \Omega^1(X) \xrightarrow{d} \Omega^2(X) \xrightarrow{*} \Omega^1(X) \simeq \Gamma(TX)

X = \mathbb{R}^3 ならば \mathrm{rot} (A_x \partial_x + A_y \partial_y + A_z \partial_z) = (\partial \times A)_x \partial_x + (\partial \times A)_y \partial_y + (\partial \times A)_z \partial_z
また、外積 \times: \Gamma(TX) \times \Gamma(TX) \to \Gamma(TX) がある
(\iota_{\dot{x}(t)} \tilde{x}(t)^{*_b} d_X A)^\sharp = \tilde{x}(t)^* \mathrm{rot}_X(A^\sharp) \times \dot{x}(t)

ryoaqryoaq

\gamma(t) = m g(\dot{x}(t), \delta x(t)) - q \langle \tilde{x}(t)^{*_b}A, \delta x(t) \rangle

-q d\theta + \gamma(t) = m g(\dot{x}(t), \delta x(t)) - q \tilde{x}(t)^*\omega_P \in \Omega^1(\tilde{x}(t)^*P)

P の自明化に依らず、P が自明でなくても定義できる
\mathbb{R} 主束 -q\tilde{x}(t)^*P (\tilde{x}(t)^*P\mathbb{R} 作用を元の -\frac{1}{q} 倍で入れたもの。-qp +' t \coloneqq -q(p + (-\frac{1}{q}t))) は -q d\theta + \gamma(t) を接続にもつ

\begin{aligned} &m \nabla_{\frac{\partial}{\partial t}} \dot{x}(t) + q \tilde{x}(t)^*(\mathrm{grad}_XV - \frac{\partial}{\partial t} A^\sharp) - q (\iota_{\dot{x}(t)} \tilde{x}(t)^{*_b} d_X A)^\sharp \\ &\qquad = m \nabla_{\frac{\partial}{\partial t}} \dot{x}(t) - q (i_t^* \iota_{\dot{x}} \tilde{x}^{*_b} d\omega_P)^\sharp \in \Gamma(\mathcal{F}, x(t)^*TX) \end{aligned}

P の自明化に依らず、P が自明でなくても定義できる
ただし i_t: \mathcal{F} \to \mathcal{F} \times \mathbb{R}

ryoaqryoaq

\mathcal{M} \coloneqq \{ x \in \mathcal{F} \mid m \nabla_{\frac{\partial}{\partial t}} \dot{x} + q \tilde{x}^*(\mathrm{grad}_XV - \frac{\partial}{\partial t} A^\sharp) - q (\iota_{\dot{x}} \tilde{x}^{*_b} d_X A)^\sharp = 0 \}
\mathcal{M} は局所的に TX と同型

ryoaqryoaq

X: 多様体
P \to X \times \mathbb{R}: 接続付き \mathbb{R} 主束
平行移動で \tau_{t_1, t_0}: P_{t_0} \xrightarrow{\sim} P_{t_1} が定義できる
P の (局所) 自明化を取り、\omega_P = d\theta + \alpha \ (\alpha \in \Omega^1(X \times \mathbb{R})) と表せば

\tau_{t_1, t_0} = * - \int_{t_0}^{t_1} \omega_P = * - \int_{t_0}^{t_1} dt V

ただし、\alpha = V dt + A \ (V \in C^\infty(X \times \mathbb{R}), A \in \Gamma(X \times \mathbb{R}, T^*X))

(局所) 自明化の取り替えで \theta' = \theta + f \ (f \in C^\infty(X \times \mathbb{R})) になったとする
\alpha' = \alpha - df
V' = V - \frac{\partial f}{\partial t}
\tau_{t_1, t_0}(x, \theta') = (x, \theta' - \int_{t_0}^{t_1} dt V'(x, t)) = (x, \theta' - f(x, t_0) - \int_{t_0}^{t_1} dt V(x, t) + f(x, t_1))

ryoaqryoaq

- \int_{t_0}^{t_1} L-q\tilde{x}(t_0)^*P \xrightarrow{\sim} -q\tilde{x}(t_1)^*P を定める
この同型は P の自明化に依らず、P が自明でなくても定義できる

射影極限を考えれば、t に依らない \mathcal{M} 上の接続付き \mathbb{R} 主束ができる

ryoaqryoaq

X: Minkowski 空間
\mathrm{Aut}(X) \simeq O(1, n - 1) \ltimes \mathbb{R}^n
X の原点を固定して、座標 (x^0 = ct, x^1, \dots, x^{n - 1}) を入れる
X 上の計量は

(dx^0)^2 - (dx^1)^2 - \cdots - (dx^{n - 1})^2 = c^2(dt)^2 - (dx^1)^2 - \cdots - (dx^{n - 1})^2

\mathcal{F} \coloneqq \{ x(\tau): \mathbb{R} \to X \mid \langle \frac{dx}{d\tau}, \frac{dx}{d\tau} \rangle \ge 0, \frac{dt}{d\tau} > 0 \}
L \in \Gamma(\mathcal{F} \times \mathbb{R}, T^*\mathbb{R})
L \coloneqq - m_0 c \left\langle \frac{dx}{d\tau}, \frac{dx}{d\tau} \right\rangle^\frac{1}{2} d\tau

で定める
v \in T_x X に対して
\begin{aligned} &v \text{ is timelike } &\Leftrightarrow \langle v, v \rangle > 0 \\ &v \text{ is lightlike } &\Leftrightarrow \langle v, v \rangle = 0 \\ &v \text{ is spacelike } &\Leftrightarrow \langle v, v \rangle < 0 \end{aligned}

H_0, H_1 を (空でない) 連結閉な timelike 超曲面 (0 でない接ベクトルが timelike) とする
\mathcal{F} の元は H_01 点で交わり、\tau_0: \mathcal{F} \to \mathbb{R} が構成できる
同様に \tau_1: \mathcal{F} \to \mathbb{R} も構成できる

https://physics.stackexchange.com/questions/727883/can-the-addition-of-two-spacelike-vectors-ever-null

https://physics.stackexchange.com/questions/428088/what-is-a-spacelike-surface-in-relativity

H_0 の相異なる 2 点の差は spacelike
H_0 \times \mathbb{R} \ni (y, x_0) \mapsto y + (x_0, 0, \dots, 0) \in X は local diffeo かつ単射かつ proper。よって、全射でもある

\int_{\tau_0}^{\tau_1} L \in C^\infty(\mathcal{F} \times \mathbb{R})

\delta \int_{\tau_0}^{\tau_1} L = - m_0 c \left( \frac{\langle \dot{x}(\tau_1), \delta x(\tau_1) \rangle}{\langle \dot{x}(\tau_1), \dot{x}(\tau_1) \rangle^\frac{1}{2}} - \frac{\langle \dot{x}(\tau_0), \delta x(\tau_0) \rangle}{\langle \dot{x}(\tau_0), \dot{x}(\tau_0) \rangle^\frac{1}{2}} \right) + m_0 c \int_{\tau_0}^{\tau_1} d\tau \frac{\langle \ddot{x}(\tau), \delta x(\tau) \rangle}{\langle \dot{x}(\tau), \dot{x}(\tau) \rangle^\frac{1}{2}}

\gamma[H] \coloneqq - m_0 c \frac{\langle \dot{x}(\tau_H), \delta x(\tau_H) \rangle}{\langle \dot{x}(\tau_H), \dot{x}(\tau_H) \rangle^\frac{1}{2}}
\mathcal{M} \coloneqq \{ x \in \mathcal{F} \mid \ddot{x} = 0 \}
H に依らない \mathcal{M} 上の接続付き自明 \mathbb{R} 主束ができる

ryoaqryoaq

T \to M: 接続付き \mathbb{R} 主束
曲率 \omega \in \Omega^2(M) は非退化だとする

この系の同型は diffeo \varphi: T \xrightarrow{\sim} T
(1) \mathbb{R} 作用を保つ
(2) \omega_T を保つ

この系の無限小の同型は \xi \in \Gamma(T, TT)
(1) \mathbb{R} 作用を保つ
(2) L_\xi \omega_T = 0

\xiVT 成分を考えると Q \in C^\infty(M) ができる
(2) を Cartan's homotopy formula で変形すると

dQ = \iota_\xi \omega

\xi \mapsto Q は単射なことがわかる

ryoaqryoaq

E \to M: ファイバー束

\mathrm{Jet}^k E \to \mathrm{Jet}^{k - 1} E には VE \to E, S^k(T^*M) \to M\mathrm{Jet}^{k - 1} E 上でのテンソル積 VE \otimes S^k(T^*M) \to \mathrm{Jet}^{k - 1} E がファイバーごとに単純推移的に作用する

\mathcal{F} \coloneqq \Gamma(M, E)
e_k: \mathcal{F} \times M \to \mathrm{Jet}^k E

\Omega_\mathrm{loc}(\mathcal{F} \times M) \coloneqq \bigcup_k e_k^*\Omega(\mathrm{Jet}^k E)

ryoaqryoaq

M \subset \mathbb{R}^m, F \subset \mathbb{R}^fE = M \times F と書けるとする
\mathrm{Jet}^k E = M \times F \times \prod_{\alpha \in \mathbb{Z}_{\ge 0}^m, 1 \le |\alpha| \le k} \mathbb{R}^f = \{ (x, y_0, y_\alpha) \}
\Omega^1(\mathrm{Jet}^k E) の元は f_i, g_{\alpha j} \in C^\infty(\mathrm{Jet}^k E) を用いて

\sum_{1 \le i \le m} f_i dx_i + \sum_{\substack{0 \le |\alpha| \le k \\ 1 \le j \le f}} g_{\alpha j} dy_{\alpha j}

と表せる
e_k\mathcal{F} \times M = \{ (\varphi, x) \} 上に引き戻すと
\sum_{1 \le i \le m} f_i(x, \partial^\alpha \varphi(x)) dx_i + \sum_{\substack{0 \le |\alpha| \le k \\ 1 \le j \le f}} g_{\alpha j}(x, \partial^\alpha \varphi(x)) D \partial^\alpha \tilde{\varphi}^j

D \partial^\alpha \tilde{\varphi}^j = d_M \partial^\alpha \varphi^j(x) + \delta \partial^\alpha \varphi^j(x) に注意

ryoaqryoaq

v_0 \in \mathrm{Jet}^k E
x_0 \coloneqq \pi(v_0) \in M
\omega_{v_0} \in T^*_{v_0}(\mathrm{Jet}^k E) が contact とは、germ s_{x_0} \in \Gamma(-, E)_{x_0} で、誘導される germ \tilde{s}_{x_0} \in \Gamma(-, \mathrm{Jet}^k E)_{x_0}\tilde{s}_{x_0}(x_0) = v_0 を満たすものに対して

\tilde{s}_{x_0}^*\omega_{v_0} = 0

が成り立つことをいう

v_0 = (x^i_0, y^{\alpha, j}_0)
\omega_{v_0} = \sum_i p_i dx^i|_{v_0} + \sum_{|\alpha| \le k, j} q_{\alpha, j} dy^{\alpha, j}|_{v_0} が contact になる条件は

\begin{aligned} &\forall s \in C^\infty(M, F)_{x_0}, (\partial^\alpha s(x_0) = y^\alpha_0 \ (|\alpha| \le k)) \Rightarrow \sum_i p_i dx^i|_{x_0} + \sum_{|\alpha| \le k, j} q_{\alpha, j} d(\partial^\alpha s^j)|_{x_0} = 0 \\ &\Leftrightarrow q_{\alpha, j} = 0 \ (|\alpha| = k) \text{ かつ } \forall i, p_i + \sum_{|\alpha| \le k - 1, j} q_{\alpha, j} y^{\alpha + e_i, j}_0 = 0 \end{aligned}

q_{\alpha, j} \ (|\alpha| \le k - 1, j) を決めるごとに p_i を調整すれば良いから、v_0 での contact cotangent vector たちは dy^{\alpha, j}|_{v_0} - \sum_i y^{\alpha + e_i, j}_0 dx^i|_{v_0} (|\alpha| \le k - 1, j) を基底に持つ
特に contact cotangent bundle は dy^{\alpha, j} - \sum_i y^{\alpha + e_i, j} dx^i (|\alpha| \le k - 1, j) を local frame に持つ
この local frame の e_k: \mathcal{F} \times M \to \mathrm{Jet}^k E による引き戻しは \delta \partial^\alpha \varphi^j(x)

ryoaqryoaq

v_0 \in \mathrm{Jet}^k E
x_0 \coloneqq \pi(v_0) \in M
X_{v_0} \in T_{v_0}(\mathrm{Jet}^k E) が Cartan とは、germ s_{x_0} \in \Gamma(-, E)_{x_0} で、誘導される germ \tilde{s}_{x_0} \in \Gamma(-, \mathrm{Jet}^k E)_{x_0}\tilde{s}_{x_0}(x_0) = v_0 を満たすものが存在して

X_{v_0} \in {\tilde{s}_{x_0}}_*(T_{x_0}M)

が成り立つことをいう

定義から、Cartan tangent vector たちの零化空間が contact cotangent vector たちで、contact cotangent vector たちの共通零点が Cartan tangent vector たち

Cartan tangent bundle の local frame は \frac{\partial}{\partial x_i} + \sum_{|\alpha| \le k - 1, j} y^{\alpha + e_i, j} \frac{\partial}{\partial y^{\alpha, j}} (1 \le i \le m), \frac{\partial}{\partial y^{\alpha, j}} (|\alpha| = k, j)

ryoaqryoaq

\pi_k: \mathrm{Jet}^k E \to M
\pi_{k, l}: \mathrm{Jet}^k E \to \mathrm{Jet}^l E \quad (k \ge l)

部分束 C^{*, k}, \pi_k^* T^*M \subset T^*(\mathrm{Jet}^k E) を contact 部分と水平部分とする

\begin{aligned} C^{*, k} &= \langle \theta^{\alpha, j} \coloneqq dy^{\alpha, j} - \sum_i y^{\alpha + e_i, j} dx^i \ (|\alpha| \le k - 1, j) \rangle \\ \pi_k^* T^*M &= \langle dx^i \rangle \end{aligned}

\varinjlim_k \Omega(\mathrm{Jet}^k E) \simeq \varinjlim_k \Gamma(\mathrm{Jet}^k E, \wedge (C^{*, k} \oplus \pi_k^* T^*M)) 上の作用素 dd = d_V + d_H と分解する (d_V は contact 部分)

\begin{aligned} (d_V + d_H) dx^i &= 0 + 0 \\ (d_V + d_H) \theta^{\alpha, j} &= 0 - \sum_i \theta^{\alpha + e_i, j} dx^i \end{aligned}

\varinjlim_k e_k^* で引き戻せば、二重複体 \Omega_\mathrm{loc}^{*, *}(\mathcal{F} \times M) ができる

複体 (\varinjlim_k \Gamma(\mathrm{Jet}^k E, {C^{*, k}}^{\otimes p} \otimes \wedge T^*M), \tilde{d_H}) を考える

(\varinjlim_k \Gamma(\mathrm{Jet}^k E, {C^{*, k}}^{\otimes p} \otimes \wedge T^*M), \tilde{d_H}) \xtofrom[\text{対称化}]{} (\varinjlim_k \Gamma(\mathrm{Jet}^k E, \wedge^p C^{*, k} \otimes \wedge T^*M), d_H)

C^{*, k + 1} / \pi_{k + 1, k}^* C^{*, k} \simeq \pi_{k + 1}^* S^k(TM) \otimes \pi_{k + 1, 0}^* V^*E
|\alpha| = k, j に対して \theta^{\alpha, j}(\frac{\partial}{\partial x})^\alpha \otimes dy^j が対応する

\varinjlim_k \Gamma(\mathrm{Jet}^k E, {C^{*, k}}^{\otimes p} \otimes \wedge T^*M)
F_N \coloneqq \varinjlim_k \bigcup_{n_1 + \cdots + n_p \le N} \Gamma(\mathrm{Jet}^k E, C^{*, n_1 + 1} \otimes \cdots \otimes C^{*, n_p + 1} \otimes \wedge T^*M) で filtration を入れると
F_N / F_{N - 1} = \varinjlim_k \bigoplus_{n_1 + \cdots + n_p = N} \Gamma(\mathrm{Jet}^k E, S^{n_1}(TM) \otimes \cdots \otimes S^{n_p}(TM) \otimes V^*E \otimes \wedge T^*M)
\tilde{d_H}(F_N) \subset F_{N + 1}
\varinjlim_k C^\infty(\mathrm{Jet}^k E) 加群の射 \tilde{d_H}: F_N / F_{N - 1} \to F_{N + 1} / F_N が誘導されるが

d_H(s_1 \otimes \cdots \otimes s_p \otimes \sigma \otimes \omega) = - \sum_{i, 1 \le l \le p} s_1 \otimes \cdots \otimes \frac{\partial}{\partial x^i} s_l \otimes \cdots \otimes s_p \otimes \sigma \otimes dx^i \wedge \omega

ryoaqryoaq

V: n 次元 \mathbb{R} 線形空間
C^q \coloneqq SV^{\otimes p} \otimes \wedge^q V^*

d(s_1 \otimes \cdots \otimes s_p \otimes \omega) \coloneqq \sum_{\substack{1 \le i \le n \\ 1 \le l \le p}} s_1 \otimes \cdots \otimes e_i s_l \otimes \cdots \otimes s_p \otimes e_i^* \wedge \omega

によって複体になる。コホモロジーは q = n 以外消える

p = 1 の場合、h(s \otimes \omega) \coloneqq \sum_i \partial_{e_i^*} s \otimes \iota_{e_i} \omega とおくと

(dh + hd)(s \otimes \omega) = (\mathrm{deg}s - \mathrm{deg}\omega + n)(s \otimes \omega)

だから
H^q(C^*) = \begin{cases} 0 &(q \ne 0) \\ \wedge^n V^* &(q = n) \end{cases}

p が一般の場合
SV^{\otimes p} \simeq S(V^{\oplus p}) を考えると 1 \otimes \cdots \otimes \overset{l}{v} \otimes \cdots \otimes 1(0, \dots, \overset{l}{v}, \dots, 0) が対応するから

d(t \otimes \omega) = \sum_i (\overbrace{e_i, \dots, e_i}^p) t \otimes e_i^* \wedge \omega

\Delta: V \xhookrightarrow{\text{diagonal}} V^{\oplus p} の補空間を固定すれば、p = 1 の場合と同じ議論ができて
H^q(C^*) = \begin{cases} 0 &(q \ne 0) \\ (S(V^{\oplus p}) / \langle \Delta(V) \rangle) \otimes \wedge^n V^* \simeq S(\mathbb{R}^{(p - 1)n}) \otimes \wedge^n V^* &(q = n) \end{cases}

ryoaqryoaq

あとは、逆にたどる

まず、F_N^q \coloneqq \varinjlim_k \bigcup_{n_1 + \cdots + n_p \le N} \Gamma(\mathrm{Jet}^k E, C^{*, n_1 + 1} \otimes \cdots \otimes C^{*, n_p + 1} \otimes \wedge T^q M) とすると (F_N^q / F_{N - 1}^q, \tilde{d_H}) のコホモロジーは q = m 以外で消える。\tilde{d_H}(F_N^q) \subset F_{N + 1}^{q + 1} に注意

(\varinjlim_k \Gamma(\mathrm{Jet}^k E, {C^{*, k}}^{\otimes p} \otimes \wedge T^*M), \tilde{d_H}) のコホモロジーも q = m 以外で消える

反対称化を考えれば、(\varinjlim_k \Gamma(\mathrm{Jet}^k E, \wedge^p C^{*, k} \otimes \wedge T^*M), d_H) のコホモロジーも q = m 以外で消える

最後に、\Omega_\mathrm{loc}^{p, *}(\mathcal{F} \times M) のコホモロジーも q = m 以外で消えることがわかる

ryoaqryoaq

(\varinjlim_k \Gamma(\mathrm{Jet}^k E, \wedge C^{*, k} \otimes \wedge^q T^*M), d_V) も気になるけど、保留

ryoaqryoaq

M: 多様体
\mathbb{Z} / 2 主束 \mathfrak{o}_M を向き付けとする

\Omega^{|-p|}_M \coloneqq \Omega^{n - p}_M \otimes \mathfrak{o}_M

ryoaqryoaq

E \to M: fiber bundle
\mathcal{F} \coloneqq \Gamma(M, E)
L \in \Omega_\mathrm{loc}^{0, |0|}(\mathcal{F} \times M)

具体例は X を Riemann 多様体として、E = X \times \mathbb{R}, \mathcal{F} = C^\infty(\mathbb{R}, X), L = \frac{1}{2} |\dot{x}|^2 dt

\delta L = \langle \dot{x}, \delta \dot{x} \rangle dt = (\frac{\partial}{\partial t} \langle \dot{x}, \delta x \rangle - \langle \ddot{x}, \delta x \rangle) dt = - d \langle \dot{x}, \delta x \rangle - \langle \ddot{x}, \delta x \rangle dt

\delta\mathcal{F} 上、dM 上、D = \delta + d\mathcal{F} \times M 上の外微分を表す

\Omega_\mathrm{loc}^{0, |0|}(\mathcal{F} \times M) \xrightarrow{\underline{D}} \Omega_\mathrm{loc}^{1, |0|}(\mathcal{F} \times M)

\underline{D}L = - \langle \ddot{x}, \delta x \rangle dt

となるように定義したい

ryoaqryoaq

\Omega_\mathrm{loc}^{1, |0|}(\mathcal{F} \times M) の元は局所的に

\sum_{j, \alpha} f_{j, \alpha}(x, \partial^\cdot \varphi(x)) \delta \partial^\alpha \varphi^j dx^1 \cdots dx^n

と表示できる。\alpha = f(x, \partial^\cdot \varphi(x)) \delta \partial^{i_1} \cdots \partial^{i_k} \varphi^j dx^1 \cdots dx^n を部分積分すると
\begin{aligned} \alpha &= (-1)^{i_1} d(f \delta \partial^{i_2} \cdots \partial^{i_k} \varphi^j dx^1 \cdots \check{dx^{i_1}} \cdots dx^n) - \partial^{i_1} f \delta \partial^{i_2} \cdots \partial^{i_k} \varphi^j dx^1 \cdots dx^n \\ &\cdots \\ &= d(\sum_{l = 1}^k (-1)^{i_l + l - 1} \partial^{i_1} \cdots \partial^{i_{l - 1}} f \delta \partial^{i_{l + 1}} \cdots \partial^{i_k} \varphi^j dx^1 \cdots \check{dx^{i_l}} \cdots dx^n) + (-1)^k \partial^{i_1} \cdots \partial^{i_k} f \delta \varphi^j dx^1 \cdots dx^n \end{aligned}

I: \Omega_\mathrm{loc}^{1, |0|}(\mathcal{F} \times M) \to \Omega_\mathrm{loc}^{1, |0|}(\mathcal{F} \times M)

I(\alpha) \coloneqq (-1)^k \partial^{i_1} \cdots \partial^{i_k} f \delta \varphi^j dx^1 \cdots dx^n

で定義する。I^2 = I だから
\begin{aligned} \Omega_\mathrm{loc}^{1, |0|}(\mathcal{F} \times M) &= \mathrm{Im} I \oplus \mathrm{Ker} I \\ &= \{ \text{linear over functions} \} \oplus d \Omega_\mathrm{loc}^{1, |-1|}(\mathcal{F} \times M) \end{aligned}

ただし、\beta \in \Omega_\mathrm{loc}^{1, |0|}(\mathcal{F} \times M) が linear over functions とは

\langle \beta, f \xi \rangle = f \langle \beta, \xi \rangle \quad (\xi \in T\mathcal{F}, f \in C^\infty(M))

が成り立つことをいう。つまり、局所的に
\beta = \sum_j g_j(x, \partial^\cdot \varphi(x)) \delta \varphi^j dx^1 \cdots dx^n

と表示できるということ

\underline{D} \coloneqq I \circ \delta

\Omega_\mathrm{loc}^{0, |0|}(\mathcal{F} \times M) \xrightarrow{\underline{D}} \Omega_\mathrm{loc}^{1, |0|}(\mathcal{F} \times M) の核と像に関しては保留

ryoaqryoaq
\underline{D} L = \delta L + d \gamma

となる \gamma \in \Omega_\mathrm{loc}^{1, |-1|}(\mathcal{F} \times M) は適当に固定する

具体例は \gamma = \langle \dot{x}, \delta x \rangle

逆に \mathcal{L} = L + \gamma \in \Omega_\mathrm{loc}^{0, |0|}(\mathcal{F} \times M) \oplus \Omega_\mathrm{loc}^{1, |-1|}(\mathcal{F} \times M)

(D \mathcal{L})^{1, |0|} = \delta L + d \gamma

が linear over functions なものが与えられたとする。I を作用させると
(D \mathcal{L})^{1, |0|} = \underline{D} L

\mathcal{M} \coloneqq \{ \varphi \in \mathcal{F} \mid \underline{D} L(\varphi) \equiv 0 \}

ただし、\underline{D} L(\varphi) \in \Omega^{|0|}(M, T^*_\varphi \mathcal{F})

\omega \coloneqq \delta \gamma \in \Omega_\mathrm{loc}^{2, |-1|}(\mathcal{F} \times M)

ryoaqryoaq

L \in \Omega_\mathrm{loc}^{0, |0|}(\mathcal{F} \times M) がオーダー 1、つまり

L = f(x, \varphi(x), \partial^i \varphi(x)) dx^1 \cdots dx^n

と表せるとする

\gamma \in \Omega_\mathrm{loc}^{1, |-1|}(\mathcal{F} \times M) に linear over functions を課すと、一意的に \gamma が定まる

\gamma = \sum_{i, j} (-1)^{i - 1} \frac{\partial f}{\partial(\partial^i y^j)} \delta \varphi^j dx^1 \cdots \check{dx^i} \cdots dx^n
ryoaqryoaq

M = \mathbb{R} \times N で、E \to M\mathbb{R} 同変束だとする

\Omega_t \coloneqq \int_{\{t\} \times N} \omega \in \Omega^2(\mathcal{M})

D \omega = 0 \ \text{on} \ \mathcal{M} \times M だから

\frac{\partial \Omega_t}{\partial t} = \int_{\{t\} \times N} \frac{\partial \omega}{\partial t} = \int_{\{t\} \times N} d_N \iota_{\partial t} \omega = 0

\delta \Omega_t = (-1)^{\mathrm{dim} N} \int_{\{t\} \times N} \delta \omega = 0

\Omega_tt に依らない閉形式 \Omega \in \Omega^2(\mathcal{M}) を定める

j \in \Omega_\mathrm{loc}^{0, |-1|}(\mathcal{F} \times M) を current という。j が conserved とは

dj = 0 \ \text{on} \ \mathcal{M} \times M

が成り立つことをいう

Q_t \coloneqq \int_{\{t\} \times N} j \in C^\infty(\mathcal{M})

j が conserved ならば、Q_tt に依らない Q \in C^\infty(\mathcal{M}) を定める

U \subset N とし、j = dt \wedge j_1 + j_2 と分解する

q_t \coloneqq \int_{\{t\} \times U} j = \int_{\{t\} \times U} j_2 \in C^\infty(\mathcal{M})


\frac{\partial q_t}{\partial t} = \int_{\{t\} \times \partial U} j_1

を満たす

ryoaqryoaq

dt + \int_{\{t\} \times N} \gamma は自明 \mathbb{R} 主束 \mathcal{M} \times \mathbb{R} 上の接続
これらの接続は - \int_{[t_0, t_1] \times N} L\mathcal{M} への制限によって同型
この系の射影極限を考えれば、t に依らない \mathcal{M} 上の接続付き自明 \mathbb{R} 主束ができる。曲率は \Omega \in \Omega^2(\mathcal{M})

ryoaqryoaq

\Gamma(\mathrm{Jet}^k E, \pi_{k, 0}^*VE) \xhookrightarrow{i_k} \Gamma(\mathcal{F}, T\mathcal{F}) がある

\Gamma_\mathrm{loc}(\mathcal{F}, T\mathcal{F}) \coloneqq \bigcup_k \mathrm{Im} i_k

局所的に

\varphi \mapsto \sum_j f_j(x, \partial^\cdot \varphi(x)) \partial y^j

と表示できるということ

ryoaqryoaq

\hat{\xi} \in \Gamma_\mathrm{loc}(T\mathcal{F})\alpha_{\hat{\xi}} \in \Omega_\mathrm{loc}^{0, |-1|}(\mathcal{F} \times M) の組が

\hat{\xi} L = d \alpha_{\hat{\xi}}

を満たす時、generalized infinitesimal symmetry という。\alpha_{\hat{\xi}} = 0 の時、\hat{\xi}L の manifest symmetry という。さらに \mathcal{L}_{\hat{\xi}} \gamma = 0 の時、\mathcal{L} = L + \gamma の manifest symmetry という

\hat{\xi} \in \Gamma_\mathrm{loc}(T\mathcal{F}), \alpha_{\hat{\xi}} \in \Omega_\mathrm{loc}^{0, |-1|}(\mathcal{F} \times M) を generalized infinitesimal symmetry とする

j_{\hat{\xi}} \coloneqq \langle \hat{\xi}, \gamma \rangle - \alpha_{\hat{\xi}} \in \Omega_\mathrm{loc}^{0, |-1|}(\mathcal{F} \times M)

とおくと

d j_{\hat{\xi}} = - \langle \hat{\xi}, d \gamma \rangle - d \alpha_{\hat{\xi}} = - \langle \hat{\xi}, d \gamma + \delta L \rangle = 0 \ \text{on} \ \mathcal{M} \times M

j_{\hat{\xi}} は conserved current

ryoaqryoaq

\delta j_{\hat{\xi}} を記述することを考える

\delta j_{\hat{\xi}} = \delta \iota_{\hat{\xi}} \gamma - \delta \alpha_{\hat{\xi}} = \mathcal{L}_{\hat{\xi}} \gamma - \iota_{\hat{\xi}} \delta \gamma - \delta \alpha_{\hat{\xi}} = - \iota_{\hat{\xi}} \omega + (\mathcal{L}_{\hat{\xi}} \gamma - \delta \alpha_{\hat{\xi}})

\mathcal{L}_{\hat{\xi}} \gamma - \delta \alpha_{\hat{\xi}} \in \Omega_\mathrm{loc}^{1, |-1|}(\mathcal{F} \times M)\mathcal{M} \times Md-closed なことを示す。すると、\mathcal{M} \times Md-exact なことがわかるから、\beta_{\hat{\xi}} \in \Omega_\mathrm{loc}^{1, |-2|}(\mathcal{F} \times M) が存在して

\begin{aligned} \mathcal{L}_{\hat{\xi}} \gamma - \delta \alpha_{\hat{\xi}} &= d \beta_{\hat{\xi}} &\ \text{on} \ \mathcal{M} \times M \\ \delta j_{\hat{\xi}} &= - \iota_{\hat{\xi}} \omega + d \beta_{\hat{\xi}} &\ \text{on} \ \mathcal{M} \times M \end{aligned}

\mathcal{L}_{\hat{\xi}} \gamma - \delta \alpha_{\hat{\xi}}\mathcal{M} \times Md-closed なことは

\begin{aligned} d(\mathcal{L}_{\hat{\xi}} \gamma - \delta \alpha_{\hat{\xi}}) &= \mathcal{L}_{\hat{\xi}} d \gamma + \delta d \alpha_{\hat{\xi}} \\ &= \mathcal{L}_{\hat{\xi}} d \gamma + \delta \mathcal{L}_{\hat{\xi}} L \\ &= \mathcal{L}_{\hat{\xi}}(d \gamma + \delta L) = 0 \ \text{on} \ \mathcal{M} \times M \end{aligned}

ryoaqryoaq

Generalized infinitesimal symmetry たちは

[(\xi_1, \alpha_1), (\xi_2, \alpha_2)] \coloneqq ([\xi_1, \xi_2], \xi_1 \alpha_2 - \xi_2 \alpha_1)

によって Lie 代数になる

ryoaqryoaq

\mathcal{F} \coloneqq C^\infty(\mathbb{R}, \mathbb{R}^r)

L \coloneqq \left\{ \frac{1}{2} |\dot{x}|^2 - \frac{1}{2} |x|^2 \right\} dt

\delta L = (\langle \dot{x}, \delta \dot{x} \rangle - \langle x, \delta x \rangle) dt = - d \langle \dot{x}, \delta x \rangle - \langle \ddot{x} + x, \delta x \rangle dt
\underline{D} L = - \langle \ddot{x} + x, \delta x \rangle dt
\gamma = \langle \dot{x}, \delta x \rangle
運動方程式は \ddot{x} = -x

A \in M_r(\mathbb{R}) は反対称、B \in M_r(\mathbb{R}) は対称とする
x \mapsto \langle A x + B \dot{x}, \partial x \rangle \in \Gamma(\mathbb{R}, x^*TX) が定めるベクトル場を \hat{\xi} \in \Gamma_\mathrm{loc}(T\mathcal{F}) とする

\hat{\xi} L = (\langle \dot{x}, A \dot{x} + B \ddot{x} \rangle - \langle x, A x + B \dot{x} \rangle) dt = (\langle \dot{x}, B \ddot{x} \rangle - \langle x, B \dot{x} \rangle) dt = d(\frac{1}{2} \langle \dot{x}, B \dot{x} \rangle - \frac{1}{2} \langle x, B x \rangle)
\alpha_{\hat{\xi}} \coloneqq \frac{1}{2} \langle \dot{x}, B \dot{x} \rangle - \frac{1}{2} \langle x, B x \rangle
とおけば、(\hat{\xi}, \alpha_{\hat{\xi}}) は generalized infinitesimal symmetry

j_{\hat{\xi}} = \langle \dot{x}, A x \rangle + \frac{1}{2} \langle \dot{x}, B \dot{x} \rangle + \frac{1}{2} \langle x, B x \rangle
x(t) = \cos t \cdot p + \sin t \cdot v \in \mathcal{M} では
j_{\hat{\xi}} = \langle v, Ap \rangle + \frac{1}{2} \langle p, Bp \rangle + \frac{1}{2} \langle v, Bv \rangle

\begin{aligned} \mathcal{L}_{\hat{\xi}} \gamma - \delta \alpha_{\hat{\xi}} &= \langle \xi \dot{x}, \delta x \rangle + \langle \dot{x}, \delta (\xi x) \rangle - \delta \alpha_{\hat{\xi}} \\ &= \langle A \dot{x} + B \ddot{x}, \delta x \rangle + \langle \dot{x}, \delta (A x + B \dot{x}) \rangle - \langle \delta \dot{x}, B \dot{x} \rangle + \langle \delta x, B x \rangle \\ &= 0 \ \text{on} \ \mathcal{M} \times \mathbb{R} \end{aligned}
\beta_{\hat{\xi}} = 0 と取れることがわかる

\begin{aligned} &\left[ \begin{pmatrix} \hat{\xi}_{AB} \\ \alpha_{AB} \end{pmatrix}, \begin{pmatrix} \hat{\xi}_{A'B'} \\ \alpha_{A'B'} \end{pmatrix} \right] \\ &= \begin{pmatrix} x \mapsto \langle [A', A] x + ([A', B] + [B', A]) \dot{x} + [B', B] \ddot{x}, \partial x \rangle \\ \frac{1}{2} \langle \dot{x}, ([A', B] + [B', A]) \dot{x} \rangle - \frac{1}{2} \langle x, ([A', B] + [B', A]) x \rangle - \langle \ddot{x} + x, [B', B] \dot{x} \rangle \end{pmatrix} \\ &= \begin{pmatrix} \hat{\xi}_{[A', A] - [B', B], [A', B] + [B', A]} \\ \alpha_{[A', A] - [B', B], [A', B] + [B', A]} \end{pmatrix} \ \text{on} \ \mathcal{M} \times \mathbb{R} \end{aligned}

Lie 代数 \{ (\hat{\xi}_{AB}, \alpha_{AB}) \}\mathcal{M} \times \mathbb{R}

(\hat{\xi}_{AB}, \alpha_{AB}) \mapsto - A - Bi

によって \mathfrak{u}(r) と同型

\zeta \coloneqq (x \mapsto - \langle \dot{x}, \partial x \rangle) + \partial_t \in \Gamma_\mathrm{loc}(T\mathcal{F}) \oplus \Gamma(T\mathbb{R}) とする

\begin{aligned} \zeta L &= 0 \\ \mathcal{L}_{\zeta} \gamma &= 0 \end{aligned}

だから、\zeta\mathcal{L} の manifest symmetry。(x \mapsto - \langle \dot{x}, \partial x \rangle, - \langle \partial_t, L \rangle) は generalized infinitesimal symmetry だから
j_\zeta = - \frac{1}{2} |\dot{x}|^2 - \frac{1}{2} |x|^2

ryoaqryoaq

M \coloneqq (\mathbb{R}^2, dx_0^2 - dx_1^2)
\mathcal{F} \coloneqq C^\infty(M, \mathbb{R})

L \coloneqq \frac{1}{2} |d\varphi|^2 dx_0 \wedge dx_1 = \frac{1}{2} \{ (\partial_0 \varphi)^2 - (\partial_1 \varphi)^2 \} dx_0 \wedge dx_1

\delta L = \{ \partial_0 \varphi \delta \partial_0 \varphi - \partial_1 \varphi \delta \partial_1 \varphi \} dx_0 \wedge dx_1 = - d(\partial_0 \varphi \delta \varphi dx_1 + \partial_1 \varphi \delta \varphi dx_0) - \{ (\partial_0 \varphi)^2 - (\partial_1 \varphi)^2 \} \delta \varphi dx_0 \wedge dx_1

\underline{D} L = - \{ (\partial_0 \varphi)^2 - (\partial_1 \varphi)^2 \} \delta \varphi dx_0 \wedge dx_1
\gamma = \partial_0 \varphi \delta \varphi dx_1 + \partial_1 \varphi \delta \varphi dx_0

\hat{\xi} \coloneqq (\varphi \mapsto - (\partial_0 \varphi) \partial y)\alpha_{\hat{\xi}}, \beta_{\hat{\xi}} を計算する

\hat{\xi} L = - \frac{1}{2} \partial_0 \{ (\partial_0 \varphi)^2 - (\partial_1 \varphi)^2 \} dx_0 \wedge dx_1 = d \left( - \frac{1}{2} \{ (\partial_0 \varphi)^2 - (\partial_1 \varphi)^2 \} dx_1 \right)

\alpha_{\hat{\xi}} = - \frac{1}{2} \{ (\partial_0 \varphi)^2 - (\partial_1 \varphi)^2 \} dx_1
\begin{aligned} \mathcal{L}_{\hat{\xi}} \gamma - \delta \alpha_{\hat{\xi}} &= - \partial_0^2 \varphi \delta \varphi dx_1 - \partial_0 \partial_1 \varphi \delta \varphi dx_0 - \partial_0 \varphi \delta \partial_0 \varphi dx_1 - \partial_1 \varphi \delta \partial_0 \varphi dx_0 - \delta \alpha_{\hat{\xi}} \\ &= - \partial_0^2 \varphi \delta \varphi dx_1 - \partial_0 \partial_1 \varphi \delta \varphi dx_0 - \partial_1 \varphi \delta \partial_1 \varphi dx_1 - \partial_1 \varphi \delta \partial_0 \varphi dx_0 \\ &= d(\partial_1 \varphi \delta \varphi) \ \text{on} \ \mathcal{M} \times M \end{aligned}

\beta_{\hat{\xi}} = \partial_1 \varphi \delta \varphi と取れる

j_{\hat{\xi}} = - \frac{1}{2} \{ (\partial_0 \varphi)^2 + (\partial_1 \varphi)^2 \} dx_1 - \partial_1 \varphi \partial_0 \varphi dx_0

ryoaqryoaq

M \coloneqq (\mathbb{R}^n, dx_0^2 - dx_1^2 - \cdots - dx_{n - 1}^2)

定義から \alpha, \beta \in \Omega^1(M)

\langle \alpha, \beta \rangle dx_0 \wedge \cdots \wedge dx_{n - 1} = \alpha \wedge * \beta

*(dx_{i_1} \wedge \cdots \wedge dx_{i_k}) = (-1)^{\#\{ l \mid x_{k_l} \ge 1 \}} \varepsilon dx_0 \wedge \cdots \check{dx_{i_1}} \cdots \check{dx_{i_k}} \cdots \wedge dx_{n - 1} \quad (\varepsilon \in \{ \pm 1 \})

(-1)^{\#\{ l \mid x_{k_l} \ge 1 \}} は計量の符号に由来する。\varepsilondx_{i_1} \wedge \cdots \wedge dx_{i_k} \wedge dx_0 \wedge \cdots \check{dx_{i_1}} \cdots \check{dx_{i_k}} \cdots \wedge dx_{n - 1} = \varepsilon dx_0 \wedge \cdots \wedge dx_{n - 1} で定義する

f \in C^\infty(M)

\begin{aligned} d * df &= d * (\sum_{\mu = 0}^{n - 1} \partial_\mu f dx_\mu) \\ &= d (\partial_0 f dx_1 \wedge \cdots \wedge dx_{n - 1} - \sum_{i = 1}^{n - 1} (-1)^i \partial_i f dx_0 \wedge \cdots \check{dx_i} \cdots \wedge dx_{n - 1}) \\ &= (\partial_0^2 - \sum_i \partial_i^2) f dx_0 \wedge \cdots \wedge dx_{n - 1} \end{aligned}

実スカラー場では \mathcal{F} \coloneqq C^\infty(M, \mathbb{R})

L \coloneqq \left\{ \frac{1}{2} |d\varphi|^2 - \frac{m^2}{2} \varphi^2 \right\} |d^n x|

\begin{aligned} \delta L &= - \langle d \delta \varphi, d \varphi \rangle |d^n x| - m^2 \varphi \delta \varphi |d^n x| \\ &= - d \delta \varphi \wedge * d \varphi - m^2 \varphi \delta \varphi |d^n x| \\ &= - d (\delta \varphi \wedge * d \varphi) - \delta \varphi \wedge d * d \varphi - m^2 \varphi \delta \varphi |d^n x| \end{aligned}
\underline{D} L = - \delta \varphi (d * d \varphi + m^2 \varphi |d^n x|)
\gamma = \delta \varphi \wedge * d \varphi
ryoaqryoaq

TODO
(1) エネルギーモーメントテンソル
(2) ゲージ対称性とネーターカレント
(3) 複素スカラー場
(4) スピノル場
(5) \mathbb{T} のゲージ場
(6) \mathbb{R}_{>0} のゲージ場
(7) \mathcal{G}_P, \mathbf{P}
(8) Pure Yang\text{--}Mills 理論
(9) 電磁気のチャージ

ryoaqryoaq

M: 多様体

\mathrm{met}(TM): U \mapsto \{ TM|_U \ \text{上の Lorentz 計量} \}
この層はファイバー束になる。\mathrm{met}(T^*M) も同様に定義する

\mathrm{met}(TM) \xrightarrow[g \mapsto \check{g}]{\sim} \mathrm{met}(T^*M)

E \to \mathrm{met}(T^*M): ファイバー束

\mathcal{F} \coloneqq \Gamma(M, E)
L \in \Omega^{0, |0|}(\mathcal{F} \times M)
\mathrm{Met}(T^*M) \coloneqq \Gamma(M, \mathrm{met}(T^*M))

\mathcal{F} \to \mathrm{Met}(T^*M) がある

\check{g} \in \mathrm{Met}(T^*M) とする

\mathcal{F} \to \mathrm{Met}(T^*M)\check{g} での fiber を \mathcal{F}_{\check{g}} とおく

\check{g}: M \to \mathrm{met}(T^*M) だが、E_{\check{g}} \coloneqq \check{g}^*(E \to \mathrm{met}(T^*M)) とおくと

\mathcal{F}_{\check{g}} = \Gamma(M, E_{\check{g}})

制限によって L_{\check{g}} \in \Omega^{0, |0|}(\mathcal{F}_{\check{g}} \times M) が誘導される

\underline{D} L \in \Omega^{0, |0|}(\mathcal{F} \times M, T^* \mathcal{F})\mathcal{F}_{\check{g}} \times M への制限は \underline{D} L_{\check{g}}

\varphi \in \mathcal{F_{\check{g}}}\underline{D} L_{\check{g}}(\varphi) = 0 を満たすとする

0 \to T_\varphi \mathcal{F}_{\check{g}} \to T_\varphi \mathcal{F} \to T_{\check{g}} \mathrm{Met}(T^*M) \to 0
0 \to T^*_{\check{g}} \mathrm{Met}(T^*M) \to T^*_\varphi \mathcal{F} \to T^*_\varphi \mathcal{F}_{\check{g}} \to 0
0 \to \Omega^{|0|}(M, T^*_{\check{g}} \mathrm{Met}(T^*M)) \to \Omega^{|0|}(M, T^*_\varphi \mathcal{F}) \to \Omega^{|0|}(M, T^*_\varphi \mathcal{F}_{\check{g}}) \to 0

\underline{D} L(\varphi) \mapsto \underline{D} L_{\check{g}}(\varphi) = 0 だから \underline{D} L(\varphi) \in \Omega^{|0|}(M, T^*_\varphi \mathcal{F})\Omega^{|0|}(M, T^*_{\check{g}} \mathrm{Met}(T^*M)) \simeq \Omega^{|0|}(M, S^2 T^*M) の元と思える。これを T_\varphi で表す

T \in \Omega^{0, |0|}(\mathcal{F} \times M, (\mathcal{F} \to \mathrm{Met}(T^*M))^* T^* \mathrm{Met}(T^*M)) \simeq \Omega^{0, |0|}(\mathcal{F} \times M, S^2 T^*M)

T(\varphi) = T_\varphi \qquad (\varphi \in \mathcal{F_{\check{g}}}, \underline{D} L_{\check{g}}(\varphi) = 0)

を満たすものが存在することがある

ryoaqryoaq

https://nigel.higson.ca/uploads/1/2/1/4/121496570/higson_-1993-_on_the_k-theory_proof_of_the_index_theorem.pdf のまとめ

K(X) 加群の射 \mathrm{Ind}_X: K(X \times T^*M) \to K(X) で以下を満たすものを構成する
(1) X に関して自然
(2) \mathrm{Ind}_{\mathrm{pt}}(\sigma_D) = \mathrm{Index}(D)
(3) \mathrm{Ind}_M(\lambda_M) = 1_{K(M)}

あとは \mathrm{ch}: K(Y) \otimes \mathbb{R} \xrightarrow{\sim} H^{\mathrm{ev}}(Y) を用いて書き直すと、指数定理

\mathrm{Index}(D) = (-1)^{\mathrm{dim} M} \int_{T^*M} \mathrm{ch}(\sigma_D) \smile \mathrm{Todd}(TM \otimes \mathbb{C})

が得られる

\mathrm{Ind}_X を構成する。まず、擬微分作用素を用いて

T^\omega: C_0(T^*M) \to \mathcal{K}(L^2(M)) \quad (\omega \in [1, \infty))

を構成する
K(C(X, C_0(T^*M))) \to K(C(X, \mathcal{K}(L^2(M))))

が誘導されるので、K(X) \simeq K(C(X)) \xrightarrow{\sim} K(C(X, \mathcal{K}(L^2(M)))) と組み合わせる

ryoaqryoaq

https://arxiv.org/pdf/math/0504555 のまとめ

埋め込み i: X \to Y に対して

i_!: K(TX) \to K(TY)

がある。K-theory の Thom 同型を使う

\mathrm{ind}_X: K(TX) \to \mathbb{Z} で以下を満たすものを 2 通りの方法で構成する
(1) X に関して自然
(2) \mathrm{ind}_{\mathrm{pt}} = \mathrm{id}_{\mathbb{Z}}
(3) i_! たちと可換

\mathrm{t\text{-}ind}_X\mathbb{R}^N への埋め込みと K-theory の Thom 同型を用いて定義する

\mathrm{a\text{-}ind}_X は擬微分作用素を用いて定義する

この 2 つが一致することから

\mathrm{Index}(D) = \mathrm{t\text{-}ind}_X(\sigma(D))

あとは \mathrm{ch}: K(Y) \otimes \mathbb{Q} \xrightarrow{\sim} H^{\mathrm{ev}}(Y, \mathbb{Q}) を用いて K-theory の Thom 同型と通常の Thom 同型を比較する

ryoaqryoaq

TODO
(1) Lie 型の微分方程式
(2) 正準変換 → 一旦スキップ
(3) symplectic reduction
(4) TQFT → 一旦スキップ
(5) 有限群の既約表現の計算 → 一旦スキップ
(A) ゲージ対称性とネーターカレント
(B) 複素スカラー場 → 一旦スキップ
(C) スピノル場
(D) \mathbb{T} のゲージ場 → 一旦スキップ
(E) \mathbb{R}_{>0} のゲージ場 → 一旦スキップ
(F) \mathcal{G}_P, \mathbf{P} → 一旦スキップ
(G) Pure Yang\text{--}Mills 理論
(H) 電磁気のチャージ → 一旦スキップ

ryoaqryoaq

\lambda: G \curvearrowright M
A: \mathbb{R} \to \mathfrak{g}
A に付随する Lie 型の微分方程式とは、x: \mathbb{R} \to M に関する方程式

\dot{x}(t) = d\lambda(A(t))(x(t))

ただし、d\lambda: \mathfrak{g} \to \Gamma(M, TM)
時間依存のベクトル場 d\lambda(A(t)) の flow とも思える

ryoaqryoaq

S: \mathbb{R} \to G が基本解とは

\begin{aligned} \dot{S}(t) &= {r_{S(t)}}_* A(t) \\ S(0) &= e \end{aligned}

を満たすことをいう。上は、左作用 G \curvearrowright G に関する A に付随する微分方程式

m \in M
S: G 上の基本解
S(t)m は初期値 x(0) = mA に付随する微分方程式の解

ryoaqryoaq

G \curvearrowright M は推移的とする
x: \mathbb{R} \to M を初期値 mA に付随する微分方程式の解とする
G 上の基本解を復元することは m の固定部分群 G_m 上の Lie 型の微分方程式を解くことに対応することを示す

\tilde{x}: \mathbb{R} \to G\tilde{x}m = x を満たす x の持ち上げとする
基本解 S: \mathbb{R} \to G が存在すれば、h: \mathbb{R} \to G_m があって

S = \tilde{x}h

h の条件を考える
\begin{aligned} {r_{h(t)}}_* {r_{\tilde{x}(t)}}_* A(t) &= \dot{S}(t) \\ &= {r_{h(t)}}_* \dot{\tilde{x}}(t) + {l_{\tilde{x}(t)}}_* \dot{h}(t) \end{aligned}

\dot{h}(t) = {r_{h(t)}}_* (\mathrm{Ad}_{\tilde{x}(t)^{-1}} A(t) - {l_{\tilde{x}(t)^{-1}}}_* \dot{\tilde{x}}(t))

B: \mathbb{R} \to \mathfrak{g}_m
B(t) \coloneqq \mathrm{Ad}_{\tilde{x}(t)^{-1}} A(t) - {l_{\tilde{x}(t)^{-1}}}_* \dot{\tilde{x}}(t)

で定義すれば、hB に付随する微分方程式の基本解

ryoaqryoaq

G が可換なら基本解は

S(t) = \exp\left(\int_0^t A(s) ds\right)

G は連結な可解群とする
G 上の Lie 型の微分方程式が与えられているとする
G / \overline{[G, G]} は可換だから、G 上の基本解を求めることは \overline{[G, G]} 上の別の Lie 型の微分方程式の基本解を求めることに帰着する
これを繰り返せば、G 上の基本解が求まる

\overline{[G, G]}[\mathfrak{g}, \mathfrak{g}] に対応する連結 Lie 部分群の閉包と一致する

ryoaqryoaq

アフィン変換群 G = GL_n(\mathbb{R}) \ltimes \mathbb{R}^nM = \mathbb{R}^n に作用する
\mathbb{R} \ni t \mapsto (A(t), b(t)) \in \mathfrak{g} に付随する微分方程式は

\dot{x} = Ax + b

\pi: G \to GL_n(\mathbb{R})
GL_n(\mathbb{R}) \curvearrowright \mathbb{R}^n に関する \pi_*(A, b) = A に付随する微分方程式は

\dot{y} = Ay

初期値 e_i \in \mathbb{R}^n の解 y_i が存在するとする
n = 1 なら、y_1(t) = \exp(\int_0^t A(s) ds)
S(t) \coloneqq (y_1(t), \dots, y_n(t)) が可逆なら、y_iGL_n(\mathbb{R}) 上への同時持ち上げ
\bigcap_{i = 1}^n G_{e_i} = \{ 1 \} だから、SGL_n(\mathbb{R}) 上の基本解
実は S(t) は常に可逆なこともわかる

\tilde{S} \coloneqq (S, 0)SG 上への持ち上げ
\tilde{S}(t) (1, \alpha(t))G 上の基本解だとすると

\dot{\alpha}(t) = S(t)^{-1} b(t)

\alpha(t) = \int_0^t S(s)^{-1} b(s) ds

元の微分方程式の初期値 v_0 の解は

x = \tilde{S}(t) (\alpha(t) + v_0)

この方法は定数変化法に対応する

ryoaqryoaq

SL_2(\mathbb{R}) \curvearrowright \mathbb{R}P^2 = \mathbb{R} \sqcup \{ \infty \} に関する Lie 型の微分方程式は Riccati の微分方程式に対応する

ryoaqryoaq

(M, \omega): symplectic 多様体
f \in C^\infty(M) に対応する Hamilton ベクトル場 X_f \in \Gamma(M, TM) とは

df = \iota_{X_f} \omega

を満たすもの。Hamilton ベクトル場全体を \mathcal{H}(M) で表すと
0 \to \mathbb{R} \to C^\infty(M) \to \mathcal{H}(M) \to 0

は完全
\mathcal{Sp}(M) \coloneqq \{ X \in \Gamma(M, TM) \mid \mathcal{L}_X \omega = 0 \} とすると
0 \to \mathcal{Sp}(M) \to \mathcal{H}(M) \to H_{\mathrm{dR}}^1(M) \to 0

も完全

\lambda: G \curvearrowright M: Lie 群の symplectic な作用
\lambda_*(X) \in \mathcal{Sp}(M) \ (X \in \mathfrak{g})

\begin{aligned} \iota_{\lambda_*([X, Y])} \omega &= - \iota_{[\lambda_*(X), \lambda_*(Y)]} \omega \\ &= - \mathcal{L}_{\lambda_*(X)} \iota_{\lambda_*(Y)} \omega + \iota_{\lambda_*(Y)} \mathcal{L}_{\lambda_*(X)} \omega \\ &= - (d \iota_{\lambda_*(X)} + \iota_{\lambda_*(X)} d) \iota_{\lambda_*(Y)} \omega \\ &= - d \iota_{\lambda_*(X)} \iota_{\lambda_*(Y)} \omega \\ &= d [\omega(\lambda_*(X), \lambda_*(Y))] \end{aligned}

\lambda_*(X) \in \mathcal{H}(M) の時、Hamilton 作用という

さらに、Hamilton 作用が Poisson 作用とは
\lambda_* の線形かつ G 同変な持ち上げ \rho: \mathfrak{g} \to C^\infty(M) が存在して

\rho([X, Y]) = \{\rho(X), \rho(Y)\} \ (\coloneqq \omega(X_{\rho(X)}, X_{\rho(Y)}) = \omega(\lambda_*(X), \lambda_*(Y)))

が成り立つことをいう

https://arxiv.org/abs/2003.14173

ryoaqryoaq

G が連結なら、\rho は自動的に G 同変になることを示す
\rho(\mathrm{Ad}_{e^{tX}} Y)(e^{tX} m) = \rho(Y)(m) を示せば良い

\begin{aligned} \frac{d}{dt} \rho(\mathrm{Ad}_{e^{tX}} Y)(e^{tX} m) = \rho([X, \mathrm{Ad}_{e^{tX}} Y])(e^{tX} m) + \langle (\iota_{\lambda_*(\mathrm{Ad}_{e^{tX}} Y)} \Omega), \lambda_*(X) \rangle (e^{tX} m) \end{aligned} = 0

symplectic な作用が Poisson 作用になる条件
(1) M が 1 連結かつコンパクト
H_{\mathrm{dR}}(M) = 0 だから Hamilton 作用
C^\infty(M)' \coloneqq \{ f \in C^\infty(M) \mid \int_M f \omega^n = 0 \}
持ち上げ \rho: \mathfrak{g} \to C^\infty(M)' が一意的に取れる
C^\infty(M)'G 不変でかつ \{*, *\} で閉じているので、\rho は条件を満たす

\int_M \{f, g\} \omega^n = \int_M X_g f \omega^n = \int_M \mathcal{L}_{X_g}(f \omega^n) = 0 \quad (f, g \in C^\infty(M))

(2) G が連結かつ半単純
[\mathfrak{g}, \mathfrak{g}] = \mathfrak{g}[\mathcal{Sp}(M), \mathcal{Sp}(M)] \subset \mathcal{H}(M) から Hamilton 作用なことがわかる
Poisson 作用に (一意的に) 持ち上がることは Whitehead's second lemma からわかる
(3) G 不変な \alpha \in \Gamma(M, T^*M) が存在して、\Omega = d\alpha
\rho(X) \coloneqq \alpha(\lambda_*(X)) とすれば良い
\begin{aligned} d\alpha(\lambda_*(X), \lambda_*(Y)) &= \lambda_*(X) \alpha(\lambda_*(Y)) - \lambda_*(Y) \alpha(\lambda_*(X)) - \alpha([\lambda_*(X), \lambda_*(Y)]) \\ &= \alpha(\lambda_*([X, Y])) \end{aligned}

ryoaqryoaq

\lambda: G \curvearrowright M: Poisson 作用
moment map \mu: M \to \mathfrak{g}^*\mu(m)(X) \coloneqq \rho(X)(m) で定義する

G \backslash M が symplectic 多様体に分解できることを見る

f: X \to Y
y \in Y が clean value とは f^{-1}(y) \subset X が injective immersion の構造を持ち、x \in f^{-1}(y) に対して

T_x f^{-1}(y) = \mathrm{Ker} f_{*, x}

が成り立つことをいう。正則値は clean value。また、A: V \to W が線形なら、すべての W の点が clean value

\xi \in \mathfrak{g}^*\mu に関して clean value と仮定する
G_{\xi} \curvearrowright \mu^{-1}(\xi)
G_{\xi} \backslash \mu^{-1}(\xi) は多様体構造をもち、\mu^{-1}(\xi) \to G_{\xi} \backslash \mu^{-1}(\xi) は submersion だと仮定する
この時、G_{\xi} \backslash \mu^{-1}(\xi) は symplectic 構造をもつ

\bar{\mu}: G \backslash M \to G \backslash \mathfrak{g}^* による \bar{\xi} \in G \backslash \mathfrak{g}^* の逆像は自然に G_{\xi} \backslash \mu^{-1}(\xi) と同一視できる

G \curvearrowright T^*G は coadjoint orbit に対応する

ryoaqryoaq

m \in \mu^{-1}(\xi)

\begin{aligned} v \in T_m(Gm)^\perp &\Leftrightarrow \omega(\lambda_*(X), v) = 0 \ (X \in \mathfrak{g}) \\ &\Leftrightarrow v\rho(X) = 0 \ (X \in \mathfrak{g}) \\ &\Leftrightarrow \mu_*(v) = 0 \\ &\Leftrightarrow v \in T_m(\mu^{-1}(\xi)) \end{aligned}

T_m(Gm)^\perp = T_m(\mu^{-1}(\xi)) がわかる
\begin{aligned} T_m(Gm) \cap T_m(\mu^{-1}(\xi)) &= \{ \lambda_*(X)(m) \mid X \in \mathfrak{g}, \mu_*(\lambda_*(X)(m)) = 0 \} \\ &= \{ \lambda_*(X)(m) \mid X \in \mathfrak{g}, (\mathrm{Ad}^*)_*(X)(\xi) = 0 \} \\ &= T_m(G_\xi m) \end{aligned}

\omega|_{\mu^{-1}(\xi)}G_{\xi} 不変かつ \pi_\xi: \mu^{-1}(\xi) \to G_{\xi} \backslash \mu^{-1}(\xi) に関して vertical
\omega_\xi \in \Omega^2(G_{\xi} \backslash \mu^{-1}(\xi)) が一意的に存在して \pi_\xi^* \omega_\xi = \omega|_{\mu^{-1}(\xi)}
\omega_\xi は非退化で閉

ryoaqryoaq

M: 多様体
ファイバー束 E \to \mathrm{met}(T^*M) の例を考える
(1) ファイバー束 E' \to M を用いて E \coloneqq \mathrm{met}(T^*M) \times_M E'
\Gamma(M, E) = \{ (g, \varphi) \mid g \text{ は } T^*M \text{ 上の Lorentz 計量}, \varphi \in \Gamma(M, E') \}
(2) m \in M, g_mT^*_m M 上の Lorentz 計量とする。g_m \in \mathrm{met}(T^*M) 上のファイバー E_{g_m}\mathrm{Spin}(T^*_m M, g_m) の既約 spinor 表現だとする
g: T^*M 上の Lorentz 計量
E_g \coloneqq (g^*E \to M) は spinor bundle

ryoaqryoaq

M: 多様体
G \curvearrowright X
\mathcal{F}_0 \coloneqq C^\infty(M, X)
L_0 \in \Omega^{0, |0|}(\mathcal{F}_0 \times M): G 不変で、オーダー 1

P: G 主束
\mathrm{con}(P) \to M: 切断が P 上の接続と対応するようなファイバー束
E \coloneqq \mathrm{con}(P) \times_M (P \times_G X)\mathrm{con}(P) 上のファイバー束
\mathcal{F} \coloneqq \Gamma(M, E) = \{ (A, \varphi) \mid A \text{ は } P \text{ 上の接続}, \varphi \in \Gamma(M, P \times_G X) \}

L \in \Omega^{0, |0|}(\mathcal{F} \times M) を構成する
A: P 上の接続
\varphi \in \Gamma(M, P \times_G X)
m \in M
C^\infty(M, X)m における 1 次の germ を構成する
p \in P_m をとる
y \in X\varphi(m) = \overline{(p, y)} で定まる
\varphi_{*, m}: T_m M \to T_{\varphi(m)} (P \times_G X) \simeq T_m M \oplus T_y X から \alpha_m: T_m M \to T_y X ができる

L(A, \varphi)(m) \coloneqq L_0((y, \alpha_m))(m)

と定義する。これは p の取り方に依らない
L\mathrm{Aut}(P) \simeq \Gamma(M, P \times_{G, \text{共役}} G) 不変
この構成を gauging the symmetry という

M の座標 (U, x_i)X の座標 (V, y_j)P の自明化をとれば

L_0 = l_0(x, f(x), \partial_i f(x)) |dx|

\gamma_0 = \sum_{i, j} (-1)^{i - 1} \frac{\partial l_0}{\partial(\partial_i y^j)} \delta f^j dx^1 \cdots \check{dx^i} \cdots dx^n

A = \omega_G + \tilde{A'} \quad (\omega_G \in \Omega^1(G, \mathfrak{g}), A' \in \Omega^1(U, \mathfrak{g}))

\varphi: U \to V とみなせば
\alpha_x(\partial_i) = \partial_i \varphi(x) + A'(\partial_i)(x)

\mathfrak{g} \to T_{\varphi(x)} X を省略していることに注意
L(A, \varphi) = l_0(x, \varphi(x), \partial_i \varphi(x) + A'(\partial_i)(x)) |dx| = L_A(\varphi)

\begin{aligned} \underline{D}L_A = &\sum_{j} \frac{\partial l_0}{\partial y^j}(x, \varphi, \partial_i \varphi + A'(\partial_i)) \delta \varphi^j |dx| \\ &- \sum_{i, j} \frac{\partial^2 l_0}{\partial_i \partial(\partial_i y^j)}(x, \varphi, \partial_i \varphi + A'(\partial_i)) \delta \varphi^j |dx| \end{aligned}

\gamma(A, \varphi) = \sum_{i, j} (-1)^{i - 1} \frac{\partial l_0}{\partial(\partial_i y^j)}(x, \varphi, \partial_i \varphi + A'(\partial_i)) \delta \varphi^j dx^1 \cdots \check{dx^i} \cdots dx^n = \gamma_A(\varphi)

ryoaqryoaq

一般化する

P: G 主束
E \to \mathrm{con}(P): \mathrm{Aut}(P) 同変ファイバー束
\mathcal{F} \coloneqq \Gamma(M, E)
\mathcal{A} \coloneqq \Gamma(M, \mathrm{con}(P))P 上の接続全体
\mathcal{F} \to \mathcal{A}
L \in \Omega^{0, |0|}(\mathcal{F} \times M)\mathrm{Aut}(P) 不変とする
A \in \mathcal{A}
E_A \coloneqq A^*(E \to \mathrm{con}(P))
A 上のファイバー \mathcal{F}_A\Gamma(M, E_A) と同型
L_A \in \Omega^{0, |0|}(\mathcal{F}_A \times M)L の制限とする

\varphi \in \mathcal{F}_AL_A に関して極値を取るとする

0 \to T^*_A \mathcal{A} \to T^*_{\varphi} \mathcal{F} \to T^*_{\varphi} \mathcal{F}_A \to 0

から
\underline{D} L(\varphi) \in \Omega^{|0|}(M, T^*_A \mathcal{A})

\begin{aligned} \Omega^{|0|}(M, T^*_A \mathcal{A}) \simeq \Gamma(M, T^*M \otimes P \times_G \mathfrak{g}^* \otimes |\wedge^\mathrm{top} T^*M|) \simeq \Omega^{|-1|}(M, P \times_G \mathfrak{g}^*) \end{aligned}

\underline{D} L(\varphi) から J_1 \in \Omega^{|-1|}(M, P \times_G \mathfrak{g}^*) が構成できる

ryoaqryoaq

\pi: P \to M: G 主束
\mathcal{A} \subset \Omega^1(P, \mathcal{g}): P 上の接続全体
\mathcal{A}\Omega^1(M, P \times_G \mathfrak{g}) が作用するアフィン空間で、\mathrm{Aut}(P) が作用する
\xi \in \mathfrak{aut}(P) \simeq \Gamma(M, P \times_G \mathfrak{g}), \nabla^A \in \mathcal{A} とすると

\xi_{\mathcal{A}, \nabla^A} = \nabla \xi \quad \text{in} \ \Omega^1(M, P \times_G \mathfrak{g})

P = G \times M として良い

\nabla^A = \pi^* \omega_G + \tilde{A} \quad \text{in} \ \Omega^1(G \times M, \mathfrak{g}) \quad (A \in \Omega^1(M, \mathfrak{g}))

\omega_G \in \Omega^1(G, \mathfrak{g})
\omega_G(X^L) \equiv X \quad (X \in \mathfrak{g})

で定まる。X^L \in \Gamma(G, TG)G \curvearrowleft G から誘導される左不変ベクトル場
また、\tilde{A} \in \Omega^1(G \times M, \mathfrak{g})
\tilde{A}(S_g, V_m) \coloneqq \mathrm{Ad}_{g^{-1}} A(V_m) \quad (S_g \in T_g G, V_m \in T_m M)

で定義する

\xi \in \Gamma(M, \mathfrak{g}) とみなす。\tilde{\xi} \in \Gamma(G \times M, \mathfrak{g})

\tilde{\xi}(g, m) \coloneqq \mathrm{Ad}_{g^{-1}} \xi(m)

で定める。\xi による G \times M 上の flow は e^{t \tilde{\xi}} \cdot

\left.\frac{d}{dt}\right|_{t = 0} (e^{t \tilde{\xi}} \cdot)^* (\pi^* \omega_G + \tilde{A}) = \widetilde{d\xi + [A, \xi]}

を示せば良い。\iota: M \ni m \mapsto (e, m) \in G \times MM 上に制限した

\left.\frac{d}{dt}\right|_{t = 0} \iota^* (e^{t \tilde{\xi}} \cdot)^* (\pi^* \omega_G + \tilde{A}) = d\xi + [A, \xi]

を示せば良い。左辺に V_m \in T_m M を代入すると
\begin{aligned} &\left.\frac{d}{dt}\right|_{t = 0} \omega_G(\left.\frac{\partial}{\partial s}\right|_{s = 0} e^{t \xi(v(s))}) + \left.\frac{d}{dt}\right|_{t = 0} \mathrm{Ad}_{e^{-t \xi(m)}} A(V_m) \\ &= \left.\frac{\partial^2}{\partial t \partial s}\right|_{t, s = 0} e^{-t \xi(m)} e^{t \xi(v(s))} - [\xi(m), A(V_m)] \\ &= \left.\frac{d}{ds}\right|_{s = 0} (- \xi(m) + \xi(v(s))) + [A, \xi](V_m) \\ &= (d\xi + [A, \xi])(V_m) \end{aligned}

ただし、v(s)V_mM 上の曲線に持ち上げたもの

ryoaqryoaq
\begin{aligned} \int \nabla^A \xi \wedge J_1 &= \int \underline{D}L (\xi_{\mathcal{F}, \varphi}) \\ &= \int \underline{D}L (\left.\frac{d}{du}\right|_{u = 0} g(u) \varphi) \\ &= \left.\frac{d}{du}\right|_{u = 0} \int L(g(u) \varphi) \\ &= 0 \end{aligned}

ただし、g(u) \mathrm{Aut}(P)\xi の flow

\int \xi \wedge \nabla^A J_1 = 0

\xi は任意だから、\nabla^A J_1 = 0

ryoaqryoaq

M: 多様体
G \curvearrowright X
\mathcal{F}_0 \coloneqq C^\infty(M, X)
L_0 \in \Omega^{0, |0|}(\mathcal{F}_0 \times M): G 不変で、オーダー 1
P: G 主束
gauging the symmetry を考える

A: P 上の接続
L_A の極値 \varphi \in \mathcal{F}_A から J_1 ができるとする
(\iota_{\xi_{\mathcal{F}_A}} \gamma_A)(\varphi) = - J_1(\xi) \quad (\xi \in \mathfrak{aut}(P))
特に、\nabla^A \xi = 0 ならば、\xi_{\mathcal{F}_A}L_A の manifest symmery で、対応する保存量の \varphi での値は - J_1(\xi) と一致する

ryoaqryoaq

\mathcal{F} \simeq \mathcal{A} \times \mathcal{F}_A

0 = (\xi_{\mathcal{F}} L)(A, \varphi) = (\iota_{\xi_{\mathcal{F}}} \delta L)(A, \varphi) = \iota_{\xi_{\mathcal{A}, A}} \delta L(\varphi) + (\iota_{\xi_{\mathcal{F}_A}} \delta L_A)(\varphi)

\iota_{\xi_{\mathcal{A}, A}} \delta L(\varphi) = \iota_{\xi_{\mathcal{A}, A}} (\underline{D}L - d\gamma)(\varphi) = \iota_{\xi_{\mathcal{A}, A}} \underline{D}L = \nabla^A \xi \wedge J_1

\gamma \in \Gamma(\mathcal{F} \times M, T^* \mathcal{F}_A \otimes \wedge^{\mathrm{top} - 1} T^*M \otimes \mathfrak{o}) なことに注意
\iota_{\xi_{\mathcal{F}_A}} \delta L_A = \iota_{\xi_{\mathcal{F}_A}} (\underline{D}L_A - d\gamma_A) = \iota_{\xi_{\mathcal{F}_A}} \underline{D}L_A + d \iota_{\xi_{\mathcal{F}_A}} \gamma_A

以上から
d \iota_{\xi_{\mathcal{F}_A}} \gamma_A(\varphi) = - \nabla^A \xi \wedge J_1 - \iota_{\xi_{\mathcal{F}_A}} \underline{D}L_A(\varphi)

(f\xi)_{\mathcal{F}_A} \alpha = f\xi_{\mathcal{F}_A} \alpha \ (f \in C^\infty(M), \alpha \in \Omega^{1, |*|}(\mathcal{F}_A \times M)) だから
df \wedge \iota_{\xi_{\mathcal{F}_A}} \gamma_A(\varphi) = - df \wedge J_1(\xi)

よって
\iota_{\xi_{\mathcal{F}_A}} \gamma_A(\varphi) = - J_1(\xi)

ryoaqryoaq

M: (V, Q) に付随する Minkowski 空間
\mathrm{Spin}(V) \curvearrowright S: spinor 表現
Clifford relation を満たす \Gamma: S^* \otimes S^* \to V, \tilde{\Gamma}: S \otimes S \to V を固定する
spinor 場とは \psi: M \to \Pi S
mass pairing M: \wedge^2 S \to \mathbb{R} を固定する

微分作用素 D_S: C^\infty(M, S) \to C^\infty(M, S^*)

C^\infty(M, S) \xrightarrow{d} C^\infty(M, V^* \otimes S) \xrightarrow{\tilde{\Gamma}} C^\infty(M, S^*)

で定める
\sigma_1(D_S) = \tilde{\Gamma} \in C^\infty(M, S^1 V \otimes \mathrm{Hom}(S, S^*))
\sigma_2(D_{S^*} D_S) = Q \in C^\infty(M, S^2 V \otimes \mathrm{End}(S))

L \coloneqq \left\{ \frac{1}{2} \psi D_S \psi - \frac{1}{2} \psi M \psi \right\} |dx|
\begin{aligned} \delta L &= \left\{ \frac{1}{2} \delta \psi D_S \psi + \frac{1}{2} \psi \tilde{\Gamma} d \delta \psi - \delta \psi M \psi \right\} |dx| \\ &= \left\{ \frac{1}{2} \delta \psi D_S \psi - \frac{1}{2} \tilde{\Gamma} d \psi \delta \psi - \delta \psi M \psi \right\} |dx| + \frac{1}{2} \tilde{\Gamma} d (\psi \delta \psi) |dx| \\ &= \delta \psi ( D_S \psi - M \psi ) |dx| - \frac{1}{2} d (\iota(\psi \tilde{\Gamma} \delta \psi) |dx|) \end{aligned}

\underline{D}L = \delta \psi ( D_S \psi - M \psi ) |dx|
\gamma = \frac{1}{2} \iota(\psi \tilde{\Gamma} \delta \psi) |dx|

ryoaqryoaq

M: Lorentz 多様体
P \to M: G 主束
\mathfrak{g} 上の G 不変内積が与えられているとする

\mathcal{F} \coloneqq \{ P \text{ 上の接続} \}
L \in \Omega^{0, |0|}(\mathcal{F} \times M)

L \coloneqq - \frac{1}{2} \langle F_A, *F_A \rangle = - \frac{1}{2} |F_A|^2 |dx|

で定義する。F_A \in \Omega^2(M, P \times_G \mathfrak{g})

\delta L = \langle \nabla \delta A, *F_A \rangle = d \langle \delta A, *F_A \rangle - \langle \delta A, \nabla *F_A \rangle

\underline{D}L = - \langle \delta A, \nabla *F_A \rangle
\gamma = - \langle \delta A, *F_A \rangle
運動方程式は \nabla *F_A = 0

ryoaqryoaq

M = M^1 \times N
G = \mathbb{R}_{>0}
\mathfrak{g} = \mathbb{R} には通常の内積を入れる

F_A = B_A - dt \wedge E_A \quad (B_A \in \Omega^2(N), E_A \in \Omega^1(N)) とすると

*F_A = *_N E_A + dt \wedge *_N B_A

d *F_A = 0
\begin{aligned} d *_N E_A &= 0 \\ d *_N B_A &= *_N \frac{\partial E_A}{\partial t} \end{aligned}

Bianchi の恒等式 d F_A = 0
\begin{aligned} d B_A &= 0 \\ d E_A &= - \frac{\partial B_A}{\partial t} \end{aligned}

ryoaqryoaq

M: n 次元 Lorentz 多様体
P \to M: G 主束
\mathfrak{g} 上の G 不変内積が与えられているとする
G \curvearrowright X: G 作用付き Riemann 多様体
V: X \to \mathbb{R}: G 不変

\mathcal{F} \coloneqq \{ (A, \varphi) \mid A \text{ は } P \text{ 上の接続}, \varphi \in \Gamma(M, P \times_G X) \}
L \in \Omega^{0, |0|}(\mathcal{F} \times M)

L \coloneqq \left( - \frac{1}{2} |F_A|^2 + \frac{1}{2} |\nabla^A \varphi|^2 - \varphi^*V \right) |dx| \eqqcolon L_1 + L_2 + L_3

で定義する

\delta L_1 = \langle \nabla^A \delta A, *F_A \rangle = d \langle \delta A, *F_A \rangle - \langle \delta A, \nabla^A *F_A \rangle
\begin{aligned} \delta L_2 &= \langle \delta A \cdot \varphi, * \nabla^A \varphi \rangle - \langle \nabla^A \delta \varphi, * \nabla^A \varphi \rangle \\ &= \langle \delta A \cdot \varphi, * \nabla^A \varphi \rangle - \langle \delta \varphi, \nabla^A * \nabla^A \varphi \rangle - d \langle \delta \varphi, * \nabla^A \varphi \rangle \\ &= \langle \delta A \cdot \varphi, * \nabla^A \varphi \rangle - \langle \delta \varphi, \square^A \varphi \rangle |dx| - d \langle \delta \varphi, * \nabla^A \varphi \rangle \end{aligned}

\square^A = (-1)^{n - 1} * \nabla^A * \nabla^A に注意

\delta L_3 = - \langle \delta \varphi, \varphi^* \mathrm{grad} V \rangle |dx|

\underline{D}L = - \langle \delta A, \nabla^A *F_A \rangle + \langle \delta A \cdot \varphi, * \nabla^A \varphi \rangle - \langle \delta \varphi, \square^A \varphi + \varphi^* \mathrm{grad} V \rangle |dx|
\gamma = - \langle \delta A, *F_A \rangle + \langle \delta \varphi, * \nabla^A \varphi \rangle

ryoaqryoaq

さらに、M は符号 (p, q) の擬 Riemann 多様体とする

\mathrm{Tr}(\nabla^{T^*M \otimes s^* V \mathcal{F}} \nabla s) = - \nabla^* \nabla s = (-1)^q * \nabla * \nabla s \quad (s \in \Gamma(M, \mathcal{F}))
これを \square^\nabla と表す

\alpha \in \Omega^1(M, s^* V \mathcal{F}), \beta \in \Gamma(s^* V^* \mathcal{F})

\begin{aligned} \int_M \langle \nabla^* \alpha, \beta \rangle |dx| &= \int_M \langle \alpha, \nabla \beta \rangle |dx| \\ &= \int_M \langle \nabla \beta \wedge * \alpha \rangle \\ &= - \int_M \langle \beta \wedge \nabla * \alpha \rangle \\ &= - \int_M \langle \beta, *^{-1} \nabla * \alpha \rangle |dx| \\ &= (-1)^{q + 1} \int_M \langle \beta, * \nabla * \alpha \rangle |dx| \end{aligned}

次に、\nabla^* \alpha = - \mathrm{Tr}(\nabla^{T^*M \otimes s^* V \mathcal{F}} \alpha) を示す
\alpha = \omega \otimes v として良い

\begin{aligned} \int_M \langle \nabla^* \alpha, \beta \rangle |dx| &= \int_M \langle \alpha, \nabla \beta \rangle |dx| \\ &= \int_M \langle \omega, \langle v, \nabla \beta \rangle \rangle |dx| \\ &= \int_M \langle \omega, d \langle v, \beta \rangle \rangle |dx| - \int_M \langle \omega, \langle \nabla v, \beta \rangle \rangle |dx| \\ &= - \int_M \mathrm{Tr}(\nabla \omega) \langle v, \beta \rangle |dx| - \int_M \langle \omega, \langle \nabla v, \beta \rangle \rangle |dx| \\ &= - \int_M \langle \mathrm{Tr}(\nabla(\omega \otimes v)), \beta \rangle |dx| \end{aligned}

ryoaqryoaq

\mathcal{K}: 加法圏
T: \mathcal{K} \xrightarrow{\sim} \mathcal{K}: 加法圏の圏同型
M[k] \coloneqq T^k(M) \quad (M \in \mathcal{K}, k \in \mathbb{Z})

L \to M \to N \to L[1]

を三角という。三角の射とは

\begin{array}{ccc} L & \to & M & \to & N & \to & L[1] \\ \Big\downarrow{\varphi} & & \Big\downarrow & & \Big\downarrow & & \Big\downarrow{\varphi[1]} \\ L' & \to & M' & \to & N' & \to & L'[1] \end{array}

特三角と呼ばれる三角たちの部分クラスがあって、以下の公理を満たす時、\mathcal{K} を前三角圏という
(TR1)

  • 特三角と同型な三角は特三角
  • L \to M は特三角に延長できる
  • L = L \to 0 \to L[1] は特三角

(TR2)

L \xrightarrow{\alpha} M \to N \xrightarrow{\gamma} L[1]

が特三角ならば
M \to N \to L[1] \xrightarrow{-\alpha[1]} M[1]

N[-1] \xrightarrow{-\gamma[-1]} L \to M \to N

も特三角
(TR3)
行が特三角の図式
\begin{array}{ccc} L & \to & M & \to & N & \to & L[1] \\ \downarrow & & \downarrow & & & & \\ L' & \to & M' & \to & N' & \to & L'[1] \end{array}

は三角の射に延長できる

ryoaqryoaq

特三角の射は 2 回合成すると 0

\begin{array}{ccc} L & \to & M & \to & N & \to & L[1] \\ \parallel && \Big\uparrow &&&& \\ L & = & L & \to & 0 & \to & L[1] \end{array}

を延長すれば良い

\mathcal{K} が前三角圏なら、\mathcal{K}^\mathrm{op} も前三角圏

\mathcal{M}: Abel 圏
加法的関手 F: \mathcal{K} \to \mathcal{M} がコホモロジー的とは
L \to M \to N \to L[1] が特三角ならば

F(L) \to F(M) \to F(N)

が完全なことをいう。シフトを考えれば、長完全列
\cdots \to F(N[k - 1]) \to F(L[k]) \to F(M[k]) \to F(N[k]) \to F(L[k + 1]) \to \cdots

ができる。M \mapsto F(M[k]) もコホモロジー的

P \in \mathcal{K}
\mathrm{Hom}(P, -): \mathcal{K} \to \mathrm{Ab}, \mathrm{Hom}(-, P): \mathcal{K}^\mathrm{op} \to \mathrm{Ab} はコホモロジー的
これを使うと、通常の 5 項補題から、特三角の射のうち 2 つが同型なら、全て同型なことがわかる
同様にして、L \xrightarrow{\alpha} M \to N \to L[1] が特三角ならば、\alpha が同型であることと N \simeq 0 は同値

ryoaqryoaq

\mathcal{M}: 加法圏

\mathcal{M} の複体のホモトピー圏 K(\mathcal{M}) が前三角圏であることを示す

M[1]^{\cdot} \coloneqq M^{\cdot + 1}, d_{M[1]} \coloneqq -d_M

\alpha: L \to M とする
\mathrm{cone}(\alpha)M^{\cdot} \oplus L^{\cdot + 1} に微分を

\begin{pmatrix} d & \alpha \\ 0 & -d \end{pmatrix}: M^{\cdot} \oplus L^{\cdot + 1} \to M^{\cdot + 1} \oplus L^{\cdot + 2}

で入れたものとする。三角
L \xrightarrow{\alpha} M \to \mathrm{cone}(\alpha) \to L[1]

と同型なものを特三角と定義する

(TR1)
\mathrm{cone}(1_L) \simeq 0 \ \text{in} \ K(\mathcal{M}) を示せば良い

\begin{pmatrix} 0 & 0 \\ 1 & 0 \end{pmatrix}: \mathrm{cone}(1_L)^{\cdot} = L^{\cdot} \oplus L^{\cdot + 1} \to L^{\cdot - 1} \oplus L^{\cdot} = \mathrm{cone}(1_L)^{\cdot - 1}

1_{\mathrm{cone}(1_L)}0 のホモトピーを与える

(TR2)
\alpha: L \to M とする

\begin{array}{ccc} M & \to & \mathrm{cone}(\alpha) & \to & L[1] & \xrightarrow{-\alpha} & M[1] \\ \parallel & & \parallel & & & & \\ M & \to & \mathrm{cone}(\alpha) & \to & \mathrm{cone}(M \to \mathrm{cone}(\alpha)) & \to & M[1] \end{array}

\begin{array}{ccc} \mathrm{cone}(\alpha)[-1] & \xrightarrow{-\mathrm{pr}_L} & L & \to & M & \to & \mathrm{cone}(\alpha) \\ \parallel & & \parallel & & & & \\ \mathrm{cone}(\alpha)[-1] & \xrightarrow{-\mathrm{pr}_L} & L & \to & \mathrm{cone}(\mathrm{cone}(\alpha)[-1] \xrightarrow{-\mathrm{pr}_L} L) & \to & \mathrm{cone}(\alpha) \end{array}

が三角の同型に延長できることを示せば良い
1 つ目は
\begin{pmatrix} 0 \\ 1 \\ -\alpha \end{pmatrix}: L[1]^{\cdot} = L^{\cdot + 1} \to M^{\cdot} \oplus L^{\cdot + 1} \oplus M^{\cdot + 1} = \mathrm{cone}(M \to \mathrm{cone}(\alpha))^{\cdot}

を考えれば良い
2 つ目は
\begin{pmatrix} \alpha & 1 & 0 \end{pmatrix}: \mathrm{cone}(\mathrm{cone}(\alpha)[-1] \xrightarrow{-\mathrm{pr}_L} L)^{\cdot} = L^{\cdot} \oplus M^{\cdot} \oplus L^{\cdot + 1} \to M^{\cdot}

を考えれば良い

(TR3)

\begin{array}{ccc} L & \xrightarrow{\alpha} & M & \to & \mathrm{cone}(\alpha) & \to & L[1] \\ \varphi \Big\downarrow & & \psi \Big\downarrow & & & & \\ L' & \xrightarrow{\alpha'} & M' & \to & \mathrm{cone}(\alpha') & \to & L'[1] \end{array}

が延長できることを示す
K(\mathcal{M}) での可換性から h: L^{\cdot} \to M'^{\cdot - 1} が存在して \psi\alpha - \alpha'\varphi = dh + hd
\begin{pmatrix} \psi & h \\ 0 & \varphi \end{pmatrix}: \mathrm{cone}(\alpha)^{\cdot} = M^{\cdot} \oplus L^{\cdot + 1} \to M'^{\cdot} \oplus L'^{\cdot + 1} = \mathrm{cone}(\alpha')^{\cdot}

とすれば良い

ryoaqryoaq

\mathcal{A}: 圏
S \subset \mathrm{mor}(\mathcal{A}): 積閉 (1_M \in S かつ s, t \in S が合成可能ならば st \in S)
S に関する \mathcal{A} の右 Ore 局所化とは、圏 \mathcal{A}_S と関手 Q: \mathcal{A} \to \mathcal{A}_S の組で以下を満たすことをいう
(L1) \mathrm{ob}(\mathcal{A}_S) = \mathrm{ob}(\mathcal{A}) で、Q は対象に関して恒等的
(L2) Q(s) \ (s \in S) は可逆
(L3) \mathcal{A}_S の射は Q(a)Q(s)^{-1} \ (a \in \mathrm{mor}(\mathcal{A}), s \in S) と表せる
(L4) Q(a) = Q(b) \ (a, b \in \mathrm{mor}(\mathcal{A})) ならば s \in S が存在して as = bs

ryoaqryoaq

a_1, a_2 \in \mathrm{mor}(\mathcal{A}), s_1, s_2 \in S とする。次は同値
(i) Q(a_1)Q(s_1)^{-1} = Q(a_2)Q(s_2)^{-1}
(ii) b_1, b_2 \in \mathrm{mor}(\mathcal{A}) が存在して、a_1 b_1 = a_2 b_2 かつ s_1 b_1 = s_2 b_2 \in S

(ii) \Rightarrow (i) は容易。(i) \Rightarrow (ii) を示す

Q(a_1) = Q(a_2)Q(s_2)^{-1}Q(s_1)

(L3), (L4) から c \in \mathrm{mor}(\mathcal{A}), t, u \in S が存在して
\begin{aligned} Q(s_2)^{-1}Q(s_1) &= Q(c)Q(t)^{-1} \\ s_1 tu &= s_2 cu \end{aligned}

Q(a_1) = Q(a_2)Q(c)Q(t)^{-1} だから、v \in S が存在して
a_1 tv = a_2 cv

d \in \mathrm{mor}(\mathcal{A}), w \in S
uw = vd

なものを見つければ良い。Q(v)^{-1}Q(u) に (L3), (L4) を適用すれば良い

ryoaqryoaq

右 Ore 局所化は次の普遍性を満たす
\mathcal{B} と関手 F: \mathcal{A} \to B があって F(s) \ (s \in S) がすべて可逆だとすると、関手 F_S: \mathcal{A}_S \to B が一意的に存在して、F_S Q = F

ryoaqryoaq

\mathcal{A}: 圏
S \subset \mathrm{mor}(\mathcal{A}): 積閉
S が right denominator set とは、以下を満たすことをいう
(D1) a \in \mathrm{mor}(\mathcal{A}), s \in S の target が同じならば、b \in \mathrm{mor}(\mathcal{A}), t \in S が存在して at = sb
(D2) sa = sb \ (a, b \in \mathrm{mor}(\mathcal{A}), s \in S) ならば、t \in S が存在して at = bt

ryoaqryoaq

\mathcal{A}: 圏
S \subset \mathrm{mor}(\mathcal{A}): 積閉
S に関する \mathcal{A} の右 Ore 局所化が (一意的に) 存在することと、S が right denominator set であることは同値

S に関する \mathcal{A} の右 Ore 局所化が存在すれば S が right denominator set になることは容易。S は right denominator set とする

\begin{array}{ccc} && L && \\ & s \swarrow && \searrow a & \\ M &&&& N \end{array}

なる図式全体を \mathrm{Span} \subset \mathrm{mor}(\mathcal{A}) \times S とする
\mathrm{Span} 上の同値関係 (a_1, s_1) \sim (a_2, s_2)

b_1, b_2 \in \mathrm{mor}(\mathcal{A}) \text{ が存在して、} a_1 b_1 = a_2 b_2 \text{ かつ } s_1 b_1 = s_2 b_2 \in S

で定義する。推移律のみ確かめる。b_1, b_2 \in \mathrm{mor}(\mathcal{A}) によって (a_1, s_1) \sim (a_2, s_2)b_2', b_3 \in \mathrm{mor}(\mathcal{A}) によって (a_2, s_2) \sim (a_3, s_3) とする
\begin{aligned} t &\coloneqq s_1 b_1 = s_2 b_2 \in S \\ t' &\coloneqq s_2 b_2' = s_3 b_3 \in S \end{aligned}

(D1) から c \in \mathrm{mor}(\mathcal{A}), u \in S が存在して
tc = t'u

s_2 b_2 c = s_2 b_2' u だから (D2) により v \in S が存在して
b_2 cv = b_2' uv

あとは
a_1(b_1 cv) = a_2 b_2 cv = a_2 b_2' uv = a_3(b_3 uv)

s_1(b_1 cv) = s_2 b_2 cv = s_2 b_2' uv = s_3(b_3 uv) \in S

から推移律が従う

\mathrm{Span} / \sim の合成を定義する。(a_1, s_1), (a_2, s_2) \in \mathrm{Span}a_1s_2 の target が同じだとする。(D1) から b \in \mathrm{mor}(\mathcal{A}), t \in S が存在して a_1 t = s_2 b

(a_2, s_2) \circ (a_1, s_1) \coloneqq (s_1 t, a_2 b)

Well-defined なことを示す。b' \in \mathrm{mor}(\mathcal{A}), t' \in Sa_1 t' = s_2 b' を満たすとする。(D1) から c \in \mathrm{mor}(\mathcal{A}), u \in S が存在して tu = t'c
s_2 (bu) = a_1 tu = a_1 t'c = s_2 (b'c)

(D2) を用いて c, u を取り直せば、bu = b'c として良い
(s_1 t, a_2 b) \sim (s_1 tu, a_2 bu) = (s_1 t'c, a_2 b'c) \sim (s_1 t', a_2 b')

\mathrm{Span} / \sim を射とする圏を \mathcal{A}_S とすれば良い

ryoaqryoaq

\mathcal{A}: 圏
S \subset \mathrm{mor}(\mathcal{A}): right denominator set
q_1, q_2: M \to N: \mathcal{A}_S の射
a_1, a_2 \in \mathrm{mor}(\mathcal{A}), s \in S が存在して

q_i = Q(a_i)Q(s)^{-1}

q_i = Q(b_i)Q(t_i)^{-1} と表す。(D1) から c \in \mathrm{mor}(\mathcal{A}), u \in S が存在して t_1 c = t_2 u \eqqcolon s

\begin{aligned} a_1 &\coloneqq a_1 c \\ a_2 &\coloneqq a_2 u \end{aligned}

ryoaqryoaq

\mathcal{K}: 前三角圏
\mathcal{M}: Abel 圏
F: \mathcal{K} \to \mathcal{M}: コホモロジー的

S \coloneqq \{ s \in \mathrm{mor}(\mathcal{K}) \mid F(s[i]) \text{ がすべて可逆} \}

とすると、S は left and right denominator set

S が left denominator set になることだけ示す
(D1)
a \in \mathrm{mor}(\mathcal{K}), s \in S は source が同じとする

\begin{array}{ccc} \mathrm{cone}(s)[-1] & \to & L & \xrightarrow{s} & M & \to & \mathrm{cone}(s) \\ \parallel && \Big\downarrow{a} &&&& \\ \mathrm{cone}(s)[-1] & \to & N & \to & \mathrm{cone}(\mathrm{cone}(s)[-1] \to N) & \to & \mathrm{cone}(s) \end{array}

の延長を考えれば良い
(D2)
as = 0 \ (a \in \mathrm{mor}(\mathcal{K}), s \in S) とする
\begin{array}{ccc} L & \xrightarrow{s} & M & \to & \mathrm{cone}(s) & \to & L[1] \\ \Big\downarrow && \Big\downarrow{a} &&&& \\ 0 & \to & N & = & N & \to & 0 \end{array}

を延長して
\begin{array}{ccccc} L & \xrightarrow{s} & M & \to & \mathrm{cone}(s) & \to & L[1] \\ \Big\downarrow && \Big\downarrow{a} && \Big\downarrow && \Big\downarrow \\ 0 & \to & N & = & N & \to & 0 \\ &&&& \Big\downarrow{t} && \\ &&&& \mathrm{cone}(\mathrm{cone}(s) \to N) && \\ &&&& \Big\downarrow && \\ &&&& \mathrm{cone}(s)[1] && \end{array}

を考えれば良い

ryoaqryoaq

\mathcal{K}: 前三角圏
\mathcal{M}: Abel 圏
F: \mathcal{K} \to \mathcal{M}: コホモロジー的

S \coloneqq \{ s \in \mathrm{mor}(\mathcal{K}) \mid F(s[i]) \text{ がすべて可逆} \}

\mathcal{K}_S は加法圏で、加法的関手 \mathcal{K}_S \to \mathcal{M} がある

\mathcal{K}_S 上の前三角圏の構造で以下を満たすものが一意的に存在する
(i) Q: \mathcal{K} \to \mathcal{K}_S は前三角圏の射
(ii) 前三角圏 \mathcal{E} と前三角圏の射 X: \mathcal{K} \to \mathcal{E}X(s) \ (s \in S) が可逆なものに対して、誘導される \mathcal{K}_S \to \mathcal{E} が前三角圏の射になる

ryoaqryoaq

S はシフトで閉じているから T_S: \mathcal{K}_S \xrightarrow{\sim} \mathcal{K}_S が誘導される
\mathcal{K} の特三角の Q による像と同型な三角を \mathcal{K}_S の特三角とする

(TR1)
a \in \mathrm{mor}(\mathcal{K}), s \in S は source が同じとする。S\emph{left} denominator set だから、以下の図式ができる

\begin{array}{ccc} L & \xrightarrow{a} & M & \to & \mathrm{cone}(a) & \to & L[1] \\ \Big\downarrow{s} && \Big\downarrow{t} &&&& \\ N & \xrightarrow{b} & P & \to & \mathrm{cone}(b) & \to & N[1] \end{array}

延長して変形すると、\mathcal{K}_S 内の図式
\begin{array}{ccc} N & \xrightarrow{Q(a)Q(s)^{-1}} & M & \to & \mathrm{cone}(a) & \to & N[1] \\ \parallel && \Big\downarrow{Q(t)} && \Big\downarrow{Q(u)} && \parallel \\ N & \xrightarrow{Q(b)} & P & \to & \mathrm{cone}(b) & \to & N[1] \end{array}

ができる

(TR2) 明らか

(TR3)
\mathcal{K}_S では可換な図式

\begin{array}{ccc} L & \to & M & \to & N & \to & L[1] \\ \Big\uparrow{s} && \Big\uparrow{t} &&&& \\ X && Y &&&& \\ \Big\downarrow{a} && \Big\downarrow{b} &&&& \\ L' & \to & M' & \to & N' & \to & L'[1] \end{array}

を考える。s, a を取り替えると、左上は可換な図式
\begin{array}{ccc} L & \to & M & \to & N & \to & L[1] \\ \Big\uparrow{s} && \Big\uparrow{t} &&&& \\ X & \xrightarrow{c} & Y &&&& \\ \Big\downarrow{a} && \Big\downarrow{b} &&&& \\ L' & \to & M' & \to & N' & \to & L'[1] \end{array}

ができる。左下も \mathcal{K}_S では可換だから、s, c, a を取り替えると、可換図式にできる。あとは延長を考えれば良い

ryoaqryoaq

\mathcal{M}: Abel 圏
\star \in \{ b, -, +, \sqcup \}

H^0: K^\star(\mathcal{M}) \to \mathcal{M}

はコホモロジー的

S^\star(\mathcal{M}) \coloneqq \{ s \in \mathrm{mor}(K^\star(\mathcal{M})) \mid H^i(s) \text{ がすべて可逆} \}

前三角圏

D^\star(\mathcal{M}) \coloneqq K^\star(\mathcal{M})_{S^\star(\mathcal{M})}

を導来圏という

ryoaqryoaq

\mathcal{K}: 前三角圏
\mathcal{K}' \subset \mathcal{K}: 充満部分圏
以下が成り立てば、\mathcal{K}' に自然な前三角圏の構造が入り、\mathcal{K}' \to \mathcal{K} は前三角圏の射
(i) M \in \mathcal{K}' \Rightarrow M[1], M[-1] \in \mathcal{K}'
(ii) \mathcal{K}' の射 L \to M に対して、特三角 L \to M \to N \to L[1]N \in \mathcal{K}' なものが存在する

ryoaqryoaq

\mathcal{K}: 前三角圏
\mathcal{M}: Abel 圏
F: \mathcal{K} \to \mathcal{M}: コホモロジー的

S \coloneqq \{ s \in \mathrm{mor}(\mathcal{K}) \mid F(s[i]) \text{ がすべて可逆} \}

\mathcal{K}' \subset \mathcal{K} は充満部分圏で以下が成り立つとする
(i) M \in \mathcal{K}' \Rightarrow M[1], M[-1] \in \mathcal{K}'
(ii) \mathcal{K}' の射 L \to M に対して、特三角 L \to M \to N \to L[1]N \in \mathcal{K}' なものが存在する

S' \coloneqq \{ s \in \mathrm{mor}(\mathcal{K}') \mid F(s[i]) \text{ がすべて可逆} \} = S \cap \mathrm{mor}(\mathcal{K}')

前三角圏の射 \mathcal{K}'_{S'} \to \mathcal{K}_S がある

\mathcal{A}: 圏
S \subset \mathrm{mor}(\mathcal{A}): right denominator set
\mathcal{A}' \subset \mathcal{A}: 充満部分圏
(r) S の元 M \to L'L' \in \mathcal{A}' ならば、S の元 N' \to MN' \in \mathcal{A}' なものが存在する
が成り立てば、S' \coloneqq S \cap \mathrm{mor}(\mathcal{A}') も right denominator set で、\mathcal{A}'_{S'} \to \mathcal{A}_S は忠実充満

L', M' \in \mathcal{A}'_{S'}
\mathcal{A}'_{S'} の射 L' \to M'\mathcal{A}_S 内で 0 だとする。L' \to M'L' \xrightarrow{Q'(s')^{-1}} N' \xrightarrow{Q'(a')} M' と分解する。\mathcal{A} の射 b: P \to N' が存在して、a'b = 0 かつ s'b \in S。(r) から s: P' \to PP' \in \mathcal{A}' なものが存在する。Q'(a')Q'(s')^{-1} = Q'(a'bs)Q'(s'bs)^{-1} = 0
次に、L' \xrightarrow{Q(s)^{-1}} N \xrightarrow{Q(a)} M' とする。t: N' \to NN' \in \mathcal{A}' なものが存在する。Q'(at)Q'(st)^{-1}\mathcal{A}_S 内で Q(a)Q(s)^{-1}

\mathcal{A}: 圏
S \subset \mathrm{mor}(\mathcal{A}): left denominator set
\mathcal{A}' \subset \mathcal{A}: 充満部分圏
(l) S の元 L' \to ML' \in \mathcal{A}' ならば、S の元 M \to N'N' \in \mathcal{A}' なものが存在する
が成り立てば、S' \coloneqq S \cap \mathrm{mor}(\mathcal{A}') も left denominator set で、\mathcal{A}'_{S'} \to \mathcal{A}_S は忠実充満

ryoaqryoaq

\mathcal{M}: Abel 圏
\star \in \{ b, -, +, \sqcup \}
D^\star(\mathcal{M}) \to D(\mathcal{M}) は忠実充満

まず、\star = + の場合、(l) が成り立つことを示す。L^+ \to M は擬同型とする。H^k(M) \simeq H^k(L^+) = 0 \ (k < k_0) とする

\mathrm{smt}^{\ge k_0}(M) \coloneqq (\cdots \to 0 \to \mathrm{Coker}(M^{k_0 - 1} \to M^{k_0}) \to M^{k_0 + 1} \to \cdots)

M \to \mathrm{smt}^{\ge k_0}(M) は擬同型

次に、\star = - の場合、(r) が成り立つことを示す。M \to L^- は擬同型とする。H^k(M) \simeq H^k(L^-) = 0 \ (k > k_0) とする

\mathrm{smt}^{\le k_0}(M) \coloneqq (\cdots \to M^{k_0 - 1} \to \mathrm{Ker}(M^{k_0} \to M^{k_0 + 1}) \to 0 \to \cdots)

\mathrm{smt}^{\le k_0}(M) \to M は擬同型

最後に、\star = b の場合は、D^b(\mathcal{M}) \to D^+(\mathcal{M}) が忠実充満なことから従う

ryoaqryoaq

\mathcal{M}: Abel 圏
D(\mathcal{M})^0 をコホモロジーが 0 次に集中している複体のなす D(\mathcal{M}) の充満部分圏とする

\mathcal{M} \to D(\mathcal{M})^0

は圏同値

H^0: D(\mathcal{M}) \to \mathcal{M} を考えれば、忠実なことがわかる。また、\mathrm{smt}^{\le 0}(\mathrm{smt}^{\ge 0}(M)) = H^0(M) から本質的全射なことがわかる
L^0, M^0 \in \mathcal{M} として、s: L^0 \to N, a: N \to M^0 とする。H^k(N) = 0 \ (k \ne 0) だから、\mathrm{smt}^{\le 0}(N) を考えれば、N^k = 0 \ (k > 0) として良い。t: N \to H^0(N), u: H^0(N) \to L^0, b: H^0(N) \to M^0 があって ut = s, bt = au は同型で

Q(a)Q(s)^{-1} = Q(b)Q(u)^{-1} = Q(bu^{-1})

ryoaqryoaq

\mathcal{M}: Abel 圏
M, N \in C(\mathcal{M})
\mathrm{Hom}_{\mathcal{M}}^k(M, N): graded object としての k 次の射たち
\mathrm{Hom}_{\mathcal{M}}^\cdot(M, N)

d\varphi^k \coloneqq d\varphi^k - (-1)^k \varphi^k d

によって \mathrm{Ab} の複体になる

I \in C(\mathcal{M}) が (C(\mathcal{M}) 内で) K-injective とは、全ての acyclic (= exact) な N \in C(\mathcal{M}) に対して、\mathrm{Hom}_{\mathcal{M}}^\cdot(N, I) も acyclic になること

ryoaqryoaq

\mathcal{M}: Abel 圏
\star \in \{ b, -, +, \sqcup \}
I \in C^\star(\mathcal{M}) に対して、以下は同値
(i) C^\star(\mathcal{M}) 内で K-injective
(ii) C(\mathcal{M}) 内で K-injective
(iii) 全ての acyclic な N \in K^\star(\mathcal{M}) に対して、\mathrm{Hom}_{K^\star(\mathcal{M})}(N, I) = 0
(iv) K^\star(\mathcal{M}) の射 \varphi: I \to M が擬同型ならば、r: M \to I が存在して r\varphi = 1_I
(v) 全ての M \in K^\star(\mathcal{M}) に対して、\mathrm{Hom}_{K^\star(\mathcal{M})}(M, I) \to \mathrm{Hom}_{D^\star(\mathcal{M})}(M, I) が同型

(i) \Rightarrow (ii)
\star = + で示す。N \in C(\mathcal{M}) は acyclic とする。I^k = 0 \ (k < k_0) とする。H^i(\mathrm{Hom}_{\mathcal{M}}^\cdot(N, I)) = H^i(\mathrm{Hom}_{\mathcal{M}}^\cdot(\mathrm{smt}^{\ge k_0 - i - 2}(N), I)) = 0
(ii) \Rightarrow (i) は明らか
(i) \Rightarrow (iii)
\mathrm{Hom}_{K^\star(\mathcal{M})}(N, I) = H^0(\mathrm{Hom}_{\mathcal{M}}^\cdot(N, I)) から従う
(iii) \Rightarrow (i)
\mathrm{Hom}_{K^\star(\mathcal{M})}(N[-i], I) = H^0(\mathrm{Hom}_{\mathcal{M}}^\cdot(N[-i], I)) = H^i(\mathrm{Hom}_{\mathcal{M}}^\cdot(N, I)) から従う
(iii) \Rightarrow (iv)

I \xrightarrow{\varphi} M \to \mathrm{cone}(\varphi) \to I[1]

\mathrm{cone}(\varphi) は acyclic。\mathrm{Hom}_{K^\star(\mathcal{M})}(-, I) を取ると
\mathrm{Hom}_{K^\star(\mathcal{M})}(M, I) \xrightarrow{\sim} \mathrm{Hom}_{K^\star(\mathcal{M})}(I, I)

(iv) \Rightarrow (iii)
\psi: N \to I とする
N \xrightarrow{\psi} I \xrightarrow{\varphi} \mathrm{cone}(\varphi) \to N[1]

\varphi は擬同型で、r: \mathrm{cone}(\varphi) \to I が存在して r\varphi = 1_I
\psi = r(\varphi\psi) = 0

(v) \Rightarrow (iii)
D^\star(\mathcal{M}) 内で N \simeq 0 だから、\mathrm{Hom}_{D^\star(\mathcal{M})}(N, I) = 0
(iv) \Rightarrow (v)
K^\star(\mathcal{M}) の射 a: M \to IQ(a) = 0 だとする。擬同型 s: I \to N が存在して sa = 0r: N \to I が存在して rs = 1_Ia = rsa = 0
a: M \to N と擬同型 s: I \to N があるとする。r: N \to I が存在して rs = 1_IQ(s)^{-1}Q(a) = Q(rs)Q(s)^{-1}Q(a) = Q(ra)

ryoaqryoaq

K^\star(\mathcal{M}) が enough K-injectives とは、全ての M \in K^\star(\mathcal{M}) に対して、K-injective な I \in K^\star(\mathcal{M}) と擬同型 M \to I が存在することをいう

ryoaqryoaq

\mathcal{M}: Abel 圏
I \in C^+(\mathcal{M}) が単射的対象からなる複体ならば、I は K-injective

N \in C^+(\mathcal{M}) は acyclic とする

\mathrm{Hom}_{K^\star(\mathcal{M})}(N[i], I[i]) = \mathrm{Hom}_{K^\star(\mathcal{M})}(N, I)

だから、I^i = 0 \ (i < 0) として、\mathrm{Hom}_{K^\star(\mathcal{M})}(N, I) = 0 を示せば良い
\varphi: N \to I を複体の射とする。h^i: N^i \to I^{i - 1}
h^i d = \varphi^{i - 1} - dh^{i - 1}

なるように構成する。h_i \coloneqq 0 \ (i < 0)h^{i - 1} まで構成できたとする
d: N^{i - 1} \to N^{i - 1} / B^{i - 1} \hookrightarrow N^i

と分解する。(\varphi^{i - 1} - dh^{i - 1})d = d(\varphi^{i - 2} - h^{i - 1}d) = ddh^{i - 2} = 0 だから
\begin{array}{ccc} N^{i - 1} & \to & N^{i - 1} / B^{i - 1} & \hookrightarrow & N^i \\ & \searrow{\varphi^{i - 1} - dh^{i - 1}} & \downarrow && \\ && I^{i - 1} && \end{array}

I^{i - 1} は単射的だから
\begin{array}{ccc} N^{i - 1} & \to & N^{i - 1} / B^{i - 1} & \hookrightarrow & N^i \\ & \searrow{\varphi^{i - 1} - dh^{i - 1}} & \downarrow & \swarrow & \\ && I^{i - 1} && \end{array}

ryoaqryoaq

\mathcal{M}: 充分単射的対象をもつ Abel 圏
K^+(\mathcal{M}) は enough K-injectives

M \in K^+(\mathcal{M}) とする。M^i = 0 \ (i < 0) として良い
単射的対象 I^{i + 1}d_I^i: I^i \to I^{i + 1}, \varphi^{i + 1}: M^{i + 1} \to I^{i + 1}
(i) d_I^i d_I^{i - 1} = 0
(ii) d_I^i \varphi^i = \varphi^{i + 1} d_M^i
(iii) \varphi^i: H^i(M) \to H^i(I) は同型
(iv) \varphi^{i + 1}: M^{i + 1} / B_M^{i + 1} \to I^{i + 1} / B_I^{i + 1} は単射
なるように構成する
i < -1 に対して、I^{i + 1} = 0, d_I^i = 0, \varphi^{i + 1} = 0 とする
I^i, d_I^{i - 1}, \varphi^i まで構成できたとする

\begin{array}{ccc} M^i & \twoheadrightarrow & M^i / B_M^i & \to & M^{i + 1} \\ \Big\downarrow && \Big\downarrow{\tiny{\text{単射}}} && \\ I^i & \twoheadrightarrow & I^i / B_I^i && \end{array}

Pushout と充分単射的対象をもつことを使うと
\begin{array}{ccc} M^i & \twoheadrightarrow & M^i / B_M^i & \to & M^{i + 1} && \\ \Big\downarrow && \Big\downarrow{\tiny{\text{単射}}} & \small{\text{pushout}} & \Big\downarrow{\tiny{\text{単射}}} & \searrow & \\ I^i & \twoheadrightarrow & I^i / B_I^i & \to & C & \hookrightarrow & I^{i + 1} \end{array}

ryoaqryoaq

\mathcal{K}, \mathcal{L}: 前三角圏
加法的関手 F: \mathcal{K} \to \mathcal{L} と自然同型 \theta: FT_\mathcal{K} \to T_\mathcal{L}F の組で特三角を保つものを前三角圏の射という
(F: \mathcal{K} \to \mathcal{L}, \theta: FT_\mathcal{K} \to T_\mathcal{L}F), (G: \mathcal{K} \to \mathcal{L}, \tau: GT_\mathcal{K} \to T_\mathcal{L}G) が前三角圏の射のとき、自然変換 \sigma: F \to G

\begin{array}{ccc} FT_\mathcal{K} & \xrightarrow{\theta} & T_\mathcal{L}F \\ \Big\downarrow{\scriptsize{\sigma T_\mathcal{K}}} && \Big\downarrow{\scriptsize{T_\mathcal{L} \sigma}} \\ GT_\mathcal{K} & \xrightarrow{\tau} & T_\mathcal{L}G \end{array}

が可換なものを三角圏の射の自然変換という

ryoaqryoaq

\mathcal{M}: Abel 圏
\star \in \{ b, -, +, \sqcup \}
K^\star(\mathcal{M})_{\mathrm{inj}} \subset K^\star(\mathcal{M}) を K-injective な複体たちのなす充満部分圏とする
K^\star(\mathcal{M})_{\mathrm{inj}} は前三角圏
I, J が K-injective な特三角 I \to J \to K \to I[1] に対して、K も K-injective なことを示す
N \in K^\star(\mathcal{M}) は acyclic とする。\mathrm{Hom}(N, -) を取れば良い

ryoaqryoaq

\mathcal{M}: 充分単射的対象をもつ Abel 圏

Resolution functor I: K^+(\mathcal{M}) \to K^+(\mathcal{M})_{\mathrm{inj}} を構成する
M \in K^+(\mathcal{M}) に対して、I(M) \in K^+(\mathcal{M})_{\mathrm{inj}} と擬同型 \iota(M): M \to I(M) を選択する。\varphi: M \to N に対して、Q(\iota(M))^{-1}Q(\varphi)Q(\iota(N)): I(M) \to I(N) は一意的に K^+(\mathcal{M}) の射 I(\varphi): I(M) \to I(N) に持ち上がる。加法的関手 I が構成できた

I は前三角圏の射になることを示す
擬同型たち I(M[1]) \xrightarrow{\iota(M[1])} M[1] \xrightarrow{\iota(M)[1]} I(M)[1] から一意的に K^+(\mathcal{M}) の射 I(M[1]) \to I(M)[1] が構成できる
また、L \to M \to N \to L[1] を特三角とすると、I(L) \to I(M) \to I(N) \to I(L)[1]D^+(\mathcal{M}) の特三角。D^+(\mathcal{M}) 内で

\begin{array}{ccc} I(L) & \xrightarrow{\alpha} & I(M) & \to & I(N) & \to & I(L)[1] \\ \parallel && \parallel && \downarrow && \parallel \\ I(L) & \xrightarrow{\alpha} & I(M) & \to & \mathrm{cone}(\alpha) & \to & I(L)[1] \end{array}

が構成できるが、5 項補題から \downarrow は同型。\mathrm{cone}(\alpha) は K-injective だから、K^+(\mathcal{M}) への一意的な持ち上げを考えれば、I(L) \to I(M) \to I(N) \to I(L)[1]K^+(\mathcal{M}) の特三角になることがわかる

ryoaqryoaq

\iota1_{K^+(\mathcal{M})} から K^+(\mathcal{M}) \xrightarrow{I} K^+(\mathcal{M})_{\mathrm{inj}} \subset K^+(\mathcal{M}) への前三角圏の射の自然変換

\begin{array}{ccc} M & \xrightarrow{\iota(M)} & I(M) \\ \Big\downarrow{\varphi} && \Big\downarrow{I(\varphi)} \\ N & \xrightarrow{\iota(N)} & I(N) \end{array}

D^+(\mathcal{M}) 内で可換だから、K^+(\mathcal{M}) 内でも可換

I は擬同型を同型に写すので、\tilde{I}: D^+(\mathcal{M}) \to K^+(\mathcal{M})_{\mathrm{inj}} を誘導する

ryoaqryoaq

E: 前三角圏
F: K^+(\mathcal{M}) \to E: 前三角圏の射
前三角圏の射 R^+F: D^+(\mathcal{M}) \to E と前三角圏の射の自然変換 \theta: F \to R^+F \circ Q の組で普遍的なものが存在する

R^+F

D^+(\mathcal{M}) \xrightarrow{\tilde{I}} K^+(\mathcal{M})_{\mathrm{inj}} \subset K^+(\mathcal{M}) \xrightarrow{F} E

で定義し、\theta \coloneqq F\iota とする
G: D^+(\mathcal{M}) \to E を前三角圏の射とし、前三角圏の射の自然変換 \tau: F \to GQ があるとする
前三角圏の射の自然変換 \sigma: R^+F \circ Q \to GQ
\begin{array}{ccc} F & \xrightarrow{\theta = F\iota} & R^+F \circ Q = FI \\ & \searrow{\tau} & \Big\downarrow{\sigma} \\ && GQ \end{array}

を満たすとする
\begin{array}{ccccccc} F(M) && \xrightarrow{F(\iota(M))} && F(I(M)) \\ & \searrow{\scriptsize{\tau(M)}} && \swarrow{\scriptsize{\sigma(M)}} & \\ && G(Q(M)) && \\ \Bigg\downarrow{\scriptsize{F(\iota(M))}} && \sim \Big\downarrow{\scriptsize{G(Q(\iota(M)))}} && \Bigg\downarrow{\scriptsize{F(I(\iota(M)))}} \\ && G(Q(I(M))) && \\ & \nearrow{\scriptsize{\tau(I(M))}} && \nwarrow{\scriptsize{\sigma(I(M))}} & \\ F(I(M)) && \underset{\sim}{\xrightarrow{F(\iota(I(M)))}} && F(I(I(M))) \end{array}

\sigma
FI \xrightarrow{FI\iota} FII \xrightarrow{F(\iota I)^{-1}} FI \xrightarrow{\tau I} GQI \xrightarrow{G(Q\iota)^{-1}} GQ

と定義するしかない

ryoaqryoaq

\mathcal{A}: 充分単射的対象をもつ Abel 圏
\mathcal{B}: Abel 圏
F: \mathcal{A} \to \mathcal{B}: 加法的関手
\mathrm{cone} の構成から

K^+(F): K^+(\mathcal{A}) \to K^+(\mathcal{B})

は前三角圏の射
Q K^+(F): K^+(\mathcal{A}) \to D^+(\mathcal{B}) の右導来関手を取ると
R^+F: D^+(\mathcal{A}) \to D^+(\mathcal{B})

ができる

M^0 \in \mathcal{A}
H^\cdot(R^+F(M^0)) は古典的な右導来関手と一致する

M \in K^+(\mathcal{A})
H^\cdot(R^+F(M)) は hypercohomology と呼ばれる

ryoaqryoaq

https://stacks.math.columbia.edu/tag/014Z

\mathcal{M}: Abel 圏
0 \to L \xrightarrow{\alpha} M \to N \to 0: C(\mathcal{M})の短完全列
q: \mathrm{cone}(\alpha) \to N

\mathrm{cone}(\alpha)^\cdot = M^\cdot \oplus L^{\cdot + 1} \to M^\cdot \to N^\cdot

で定義すると、複体の射。蛇の補題のような議論から、q は擬同型なことがわかる
L \to M \to N \xrightarrow{Q(\mathrm{cone}(\alpha) \to L[1])Q(q)^{-1}} L[1]

D(\mathcal{M}) の特三角

ryoaqryoaq

\mathcal{A}: 充分単射的対象をもつ Abel 圏
\mathcal{B}: Abel 圏
F: \mathcal{A} \to \mathcal{B}: 左完全関手

A \in C^+(\mathcal{A})

H^p(R^+F(A^i)) = 0 \ (p > 0)

を満たすとする。R^+F(A) = F(A) を示す

A の Cartan\text{--}Eilenberg resolution

A^\cdot \to I^{\cdot, \star}

を取る。全複体を取ると
A \to \mathrm{Tot}(I^{\cdot, \star})

は K-injective resolution で
R^+F(A) = \mathrm{Tot}(F(I^{\cdot, \star}))

F(A) \to \mathrm{Tot}(F(I^{\cdot, \star}))

がある。コホモロジーを取ると、スペクトル系列から、両辺とも H(F(A))

ryoaqryoaq

X: パラコンパクト Hausdorff 空間
\mathrm{Sh}(X): X 上の Abel 群の層がなす圏
\mathcal{U} = \{ U_i \}_{i \in I}: X の開被覆
C_{\mathcal{U}}: \mathrm{Sh}(X) \to C^+(\mathrm{Ab}) がある
F \in \mathrm{Sh}(X) に対して

C_{\mathcal{U}}(F)^n \coloneqq \prod_{(i_0, \dots, i_n) \in I^{n + 1}} F(U_{i_0, \dots, i_n}) = \{ (s_{i_0, \dots, i_n})_{(i_0, \dots, i_n) \in I^{n + 1}} \}

ただし U_{i_0, \dots, i_n} \coloneqq U_{i_0} \cap \cdots \cap U_{i_n}
d((s_{i_0, \dots, i_n})_{(i_0, \dots, i_n) \in I^{n + 1}}) \coloneqq \left( \sum_{k = 0}^{n + 1} (-1)^k s_{j_0, \dots, \check{j_k}, \dots, j_{n + 1}} \right)_{(j_0, \dots, j_{n + 1}) \in I^{n + 2}}

\mathcal{V} = \{ V_j \}_{j \in J}\mathcal{U} の細分とは \alpha: I \to J があって U_i \subset V_{\alpha(i)} となること
細分に関する余極限を取ると
C: \mathrm{Sh}(X) \to C^+(\mathrm{Ab}) ができる
H^n(R^+\Gamma(F)) \xrightarrow{\sim} H^n(C(F)) \simeq \varinjlim_{\mathcal{U}} H^n(C_{\mathcal{U}}(F))

が知られている

M \in K^+(\mathrm{Sh}(X))
\mathrm{Tot}(C(M^\cdot)^\star) = R^+\Gamma(M) \in D^+(\mathrm{Ab}) を示したい
M^\cdot \to J^{\cdot, \star} を Cartan\text{--}Eilenberg resolution とする
M^\cdot \to \mathrm{Tot}(J^{\cdot, \star}) は K-injective resolution だから

R^+\Gamma(M) = \mathrm{Tot}(\Gamma(J^{\cdot, \star}))

\begin{aligned} \mathrm{Tot}(C(M^\cdot)^\star) &\to \mathrm{Tot}(C(\mathrm{Tot}(J^{\cdot, -}))^\star) &= \mathrm{Tot}(C(J^{\cdot, -})^\star) &= \mathrm{Tot}(\mathrm{Tot}^{-, \star}(C(J^{\cdot, -})^\star)) \\ \mathrm{Tot}(\Gamma(J^{\cdot, -})) &&\to \mathrm{Tot}(C(J^{\cdot, -})^\star) &= \mathrm{Tot}(\mathrm{Tot}^{-, \star}(C(J^{\cdot, -})^\star)) \end{aligned}

がそれぞれ擬同型を示せば良い
1 つ目は、\star に関してコホモロジーを取ると、E_1 term がいずれも H^\star(R^+\Gamma(M^\cdot))
2 つ目は、- に関してコホモロジーを取ると、E_1 term がいずれも H^-(R^+\Gamma(M^\cdot))

ryoaqryoaq

X: n 次元パラコンパクト多様体
\underline{\mathbb{R}}_X \xrightarrow{\sim}_q (\Omega^0 \to \cdots \to \Omega^n)
\Omega^i は fine だから、R^+\Gamma(\underline{\mathbb{R}}_X) = (\Omega^0(X) \to \cdots \to \Omega^n(X))

ryoaqryoaq

https://webhomes.maths.ed.ac.uk/~v1ranick/papers/iversensheaves.pdf

X: パラコンパクト Hausdorff かつ局所可縮な位相空間 (hereditarily paracompact は仮定しない)
R: 可換環
S^n(U) \coloneqq \{ U \text{ 内の } R \text{ 係数の特異 } n \text{ コチェイン} \}
S^n は前層
\widetilde{S^n}S^n の層化とする
前層の射 S^n \xrightarrow{\iota} \widetilde{S^n} の核を S_0^n とする
S_0^n(U) = \{ \varphi \in S^n(U) \mid \varphi_x = 0 \ (x \in U) \}
X のパラコンパクト性から

0 \to S_0^n(X) \to S^n(X) \xrightarrow{\iota} \widetilde{S^n}(X) \to 0 \qquad \text{(*)}

が完全になることが知られている。前層 S^n が貼り合わせの存在のみ満たすことを使う

\widetilde{S^n} が soft を示す
A \subset X: 閉
A \subset U \overset{\text{open}}{\subset} X, \alpha \in \widetilde{S^n}(U)
X の開集合 WA \subset W \subset \overline{W} \subset U なものが取れる
(*) を \overline{W} に適用すると、t \in S^n(X) が存在して

\alpha|_W = \iota(t)|_W

X が局所可縮なことから

0 \to \underline{R}_X \to \widetilde{S^0} \to \widetilde{S^1} \to \cdots

は stalk ごとに完全で
\underline{R}_X \xrightarrow{\sim}_q (\widetilde{S^0} \to \widetilde{S^1} \to \cdots)

R^+\Gamma(\underline{R}_X) = \widetilde{S^\cdot}(X)
H^k(R^+\Gamma(\underline{R}_X)) = H^k(\widetilde{S^\cdot}(X)) = H^k(S^\cdot(X))
右の等号は、(*) から誘導される長完全列と H^k(S_0^\cdot(X)) = 0 から従う

H^k(S_0^\cdot(X)) = 0 を示す
\mathcal{U} = \{ U_i \}_{i \in I}X の開被覆とする
S_{\mathcal{U}}^n \coloneqq \mathrm{Hom}_{\mathbb{Z}}(\sum_{i \in I} S_n(U_i), R)
r_{\mathcal{U}}: S^\cdot(X) \twoheadrightarrow S_{\mathcal{U}}^{\cdot} は chain homotpy 同値
\varinjlim_{\mathcal{U}} \mathrm{Ker}(r_{\mathcal{U}}) = S_0^\cdot(X) だから

H^k(S_0^\cdot(X)) = H^k(\varinjlim_{\mathcal{U}} \mathrm{Ker}(r_{\mathcal{U}})) = \varinjlim_{\mathcal{U}} H^k(\mathrm{Ker}(r_{\mathcal{U}})) = 0

ryoaqryoaq

X: パラコンパクト Hausdorff かつ局所可縮な位相空間
L \to X: 複素線束
\mathcal{U} = \{ U_i \}_{i \in I}: X の開被覆で、L|_{U_i} は自明
自明化 \varphi_i: U_i \times \mathbb{C} \xrightarrow{\sim} L|_{U_i} を固定する
g_{ij}: U_{ij} \to \mathbb{C}^\times: U_j から U_i への変換関数
\overline{(g_{ij})} \in H_{\mathcal{U}}(\underline{\mathbb{C}^\times}_X)^1\mathcal{U} に関する余極限は H^1(X, \underline{\mathbb{C}^\times}_X) の元を定める

0 \to \underline{\mathbb{Z}}_X \xrightarrow{2\pi i \cdot} \underline{\mathbb{C}}_X \xrightarrow{\mathrm{exp}} \underline{\mathbb{C}^\times}_X \to 0

\underline{\mathbb{C}}_X は fine だから
H^1(X, \underline{\mathbb{C}^\times}_X) \xrightarrow{\sim} H^2(X, \underline{\mathbb{Z}}_X)

H^2(X, \underline{\mathbb{Z}}_X) の元 \tilde{c_1}(L) が定まるが、L の first chern class の -1 倍と一致する
U_{i_0, \dots, i_n} が全て可縮だとする
g_{ij} ごとに \mathrm{log} \, g_{ij} を固定できる
c_{ijk} \coloneqq \frac{1}{2\pi i} (\mathrm{log} \, g_{jk} - \mathrm{log} \, g_{ik} + \mathrm{log} \, g_{ij}) \in Z_{\mathcal{U}}(\underline{\mathbb{Z}}_X)^2

\tilde{c_1}(L) を定める

ryoaqryoaq

\gamma^1 \to \mathbb{C}P^1 に対して \tilde{c_1}(\gamma^1) = - c_1(\gamma^1) を示せば良い
\mathbb{C}P^1 を 2 つの \mathbb{C} で被覆する。\mathbb{C}_z, \mathbb{C}_w と表す
U_1 \coloneqq \{ re^{i\theta} + e^{\frac{4\pi i}{3}} \in \mathbb{C}_z \mid r \in \mathbb{R}_{> 0}, 0 < \theta < \frac{2\pi}{3} \}
U_2 \coloneqq \{ re^{i\theta} + 1 \in \mathbb{C}_z \mid r \in \mathbb{R}_{> 0}, \frac{2\pi}{3} < \theta < \frac{4\pi}{3} \}
U_3 \coloneqq \{ re^{i\theta} + e^{\frac{2\pi i}{3}} \in \mathbb{C}_z \mid r \in \mathbb{R}_{> 0}, \frac{4\pi}{3} < \theta < 2\pi \}
U_4 \coloneqq \{ w \in \mathbb{C}_w \mid |w| < 1 \}
とすると、U_{i_0, \dots, i_n} は全て可縮
g_{ij} = g_{ji}^{-1}
g_{ii} = 1
g_{12} = g_{23} = g_{13} = 1
g_{14} = g_{24} = g_{34} = \frac{1}{z}
c_{\sigma(i)\sigma(j)\sigma(k)} = \mathrm{sgn}(\sigma) c_{ijk}
c_{iij} = 0
c_{123} = c_{234} = c_{412} = 0
c_{341} = -1

ryoaqryoaq

X: パラコンパクト Hausdorff 空間
\mathcal{F}: X 上の Abel 群の層
\mathcal{U} = \{ U_i \}_{i \in I}: X の開被覆
H^k(U_{i_0, \dots, i_n}, \mathcal{F}|_{U_{i_0, \dots, i_n}}) = 0 \ (k > 0) ならば

R^+\Gamma\mathcal{F} = C_{\mathcal{U}}(\mathcal{F})

0 \to \mathcal{F} \to I^0 \to \cdots を単射的分解とする
C_{\mathcal{U}}(I^\cdot)^\star\cdot に関しても \star に関しても完全なことがわかれば

R^+\Gamma\mathcal{F} = \mathrm{Tot}(C_{\mathcal{U}}(I^\cdot)^\star) = C_{\mathcal{U}}(\mathcal{F})

\star に関して完全なことは、I^\cdot が単射的前層なことから従う
\cdot に関して完全なことを示す
0 \to \mathcal{F}|_{U_{i_0, \dots, i_n}} \to I^0|_{U_{i_0, \dots, i_n}} \to \cdots

は単射的分解だから、仮定から
0 \to \mathcal{F}(U_{i_0, \dots, i_n}) \to I^0(U_{i_0, \dots, i_n}) \to \cdots

も完全

ryoaqryoaq

X: hereditarily paracompact Hausdorff かつ局所可縮な位相空間 (距離化可能なら十分)
R: 可換環
\mathcal{U} = \{ U_i \}_{i \in I}: X の good covering

\begin{array}{ccccccc} C_{\mathcal{U}}(\widetilde{S^0})^0 & \xrightarrow{\delta} & C_{\mathcal{U}}(\widetilde{S^1})^0 & \xrightarrow{\delta} & C_{\mathcal{U}}(\widetilde{S^2})^0 & \xrightarrow{\delta} & \cdots \\ \downarrow{\scriptsize{d}} && \downarrow{\scriptsize{d}} && \downarrow{\scriptsize{d}} && \\ C_{\mathcal{U}}(\widetilde{S^0})^1 & \xrightarrow{\delta} & C_{\mathcal{U}}(\widetilde{S^1})^1 & \xrightarrow{\delta} & C_{\mathcal{U}}(\widetilde{S^2})^1 & \xrightarrow{\delta} & \cdots \\ \downarrow{\scriptsize{d}} && \downarrow{\scriptsize{d}} && \downarrow{\scriptsize{d}} && \\ C_{\mathcal{U}}(\widetilde{S^0})^2 & \xrightarrow{\delta} & C_{\mathcal{U}}(\widetilde{S^1})^2 & \xrightarrow{\delta} & C_{\mathcal{U}}(\widetilde{S^2})^2 & \xrightarrow{\delta} & \cdots \\ \downarrow{\scriptsize{d}} && \downarrow{\scriptsize{d}} && \downarrow{\scriptsize{d}} && \\ \vdots && \vdots && \vdots && \end{array}

行に関しても列に関しても完全だから、D^+(R\text{-Mod}) 内で
C_{\mathcal{U}}(\underline{R}_X)^\star = \mathrm{Tot}(C_{\mathcal{U}}(\widetilde{S^\cdot})^\star) = \widetilde{S^\cdot}(X)

また、0 \to S_0^n \to S^n \xrightarrow{\iota} \widetilde{S^n} \to 0 は前層の完全列

ryoaqryoaq

c \in Z_{\mathcal{U}}(\widetilde{S^0})^2\delta c = 0
a \in C_{\mathcal{U}}(\widetilde{S^0})^1
a_{ij} = - a_{ji}
a_{ij} = 0 \ (\{ i, j \} \ne \{ 1, 3 \})
a_{13}(z) = \begin{cases} -1 &\quad (|z| > 1) \\ 0 &\quad (|z| \le 1) \end{cases}
で定義すると、da = c
(\delta a)_{ij} = - (\delta a)_{ji}
(\delta a)_{ij} = 0 \ (\{ i, j \} \ne \{ 1, 3 \})
(\delta a)_{13}(\gamma) = a_{13}(\gamma(1)) - a_{13}(\gamma(0))
b \in C_{\mathcal{U}}(\widetilde{S^1})^0
b_i = 0 \ (i \ne 1)
b_1(\gamma) = \begin{cases} 1 &\quad (|\gamma(0)| \le 1, \gamma(1) \in U_{134}) \\ -1 &\quad (\gamma(0) \in U_{134}, |\gamma(1)| \le 1) \\ 0 &\quad (\text{otherwise}) \end{cases}
で定義すると、db = \delta a
(\delta b)_i = 0 \ (i \ne 1)
(\delta b)_1(\sigma) = \begin{cases} \mathrm{sgn}(\tau) &\quad (|\sigma_{\tau(0)}| \le 1, \sigma_{\tau(1)} \in U_{134}, \sigma_{\tau(2)} \in U_{14} \setminus U_3) \\ 0 &\quad (\text{otherwise}) \end{cases}
v \in \widetilde{S^2}(X)
v(\sigma) = \begin{cases} \mathrm{sgn}(\tau) &\quad (|\sigma_{\tau(0)}| \le 1, \sigma_{\tau(1)} \in U_{134}, \sigma_{\tau(2)} \in U_{14} \setminus U_3) \\ 0 &\quad (\text{otherwise}) \end{cases}
で定義すると、(\widetilde{S^2}(X) \to Z_{\mathcal{U}}(\widetilde{S^2})^0)v = \delta b
C_{\mathcal{U}}(\underline{R}_X)^\star = \mathrm{Tot}(C_{\mathcal{U}}(\widetilde{S^\cdot})^\star) = \widetilde{S^\cdot}(X) \ \text{in} \ D^+(R\text{-Mod})c \in H^2(C_{\mathcal{U}}(\underline{R}_X)^\star)v \in H^2(\widetilde{S^\cdot}(X)) が対応する
v\mathbb{C}P^1 の基本類とのペアリングは 1